Как рассчитать мощность электрического тока: Как найти мощность электрического тока

Содержание

формула, расчёт силы тока, напряжения и сопротивления

Безаварийная работа устройства зависит от соответствия технических характеристик прибора нормам питающей сети. Зная напряжение, сопротивление и силу тока в цепи, электрик поймёт, как найти мощность. Формула расчёта важного параметра зависит от свойств сети, в которую подключается потребитель.

Труд электричества

Механические устройства и электрические приборы предназначены для выполнения работы. Согласно второму закону Ньютона, кинетическая энергия, которая воздействует на материальную точку в течение определённого промежутка времени, совершает полезное действие. В электродинамике поле, созданное разностью потенциалов, переносит заряды на участке электрической цепи.

Объём, производимой током работы, зависит от интенсивности электричества. В середине XIX века Д. П. Джоуль и Э. Х. Ленц решали одинаковую проблему. В проводимых опытах кусок проволоки с высоким сопротивлением разогревался, когда через него пропускался ток.

Учёных интересовал вопрос, как вычислить мощность цепи. Для понимания процесса, происходящего в проводнике, следует ввести следующие определения:

  • P — мощность.
  • A — работа, совершаемая зарядом в электрической цепи.
  • U — падение напряжения в проводнике.
  • I — сила тока.
  • Q — количество электрических зарядов, переносимых в единицу времени.

Мощность — это работа, производимая током в проводнике за какой-то временной период. Утверждение описывает формула: P = A ∕ ∆t.

На участке цепи разность потенциалов в точках a и b совершает работу по перемещению электрических зарядов, которая определяется уравнением: A = U ∙ Q. Ток представляет собой суммарный заряд, прошедший в проводнике за единицу времени, что математически выражается соотношением: U ∙ I = Q ∕ ∆t. После преобразований получается формула мощности электрического тока: P = A ∕ ∆t = U ∙ Q ∕ ∆t = U ∙ I. Можно утверждать, что в цепи проводится работа, которая зависит от мощности, определяемой током и напряжением на контактах подключённого электрического устройства.

Производительность постоянного тока

В линейной цепи без конденсаторов и катушек индуктивности соблюдается закон Ома. Немецкий учёный обнаружил взаимосвязь тока и напряжения от сопротивления цепи. Открытие выражается уравнением: I = U ∕ R. При известном значении сопротивления нагрузки мощность вычисляется двумя способами: P = I ² ∙ R или P = U ² ∕ R.

Если ток в цепи течёт от плюса к минусу, то энергия сети поглощается потребителем. Такой процесс проистекает при зарядке аккумуляторной батареи. Если движение тока совершается в противоположном направлении, то мощность отдаётся в электрическую цепь. Так происходит в случае питания сети от работающего генератора.

Мощность переменной сети

Расчёт переменных цепей отличается от вычисления параметра производительности в линии постоянного тока. Это связано с тем, что напряжение и ток изменяются во времени и по направлению.

В цепи со сдвигом фаз тока и напряжения, рассматриваются следующие виды мощности:

  1. Активная.
  2. Реактивная.
  3. Полная.

Активный компонент

Активная часть полезной мощности учитывает скорость невозвратного преобразования электричества в тепловую или магнитную энергию. В линии тока с одной фазой активная составляющая вычисляется по формуле: P = U ∙ I ∙ cos ϕ.

В международной системе единиц СИ величина производительности измеряется в ваттах. Угол ϕ определяет смещение напряжения по отношению к току. В трёхфазной цепи активная часть складывается из суммы мощностей каждой отдельной фазы.

Реверсивные потери

Для работы конденсаторов, катушек индуктивности, обмоток электродвигателей затрачивается сила сети. Из-за физических свойств таких устройств энергия, которая определяется реактивной мощностью, возвращается в цепь. Величина отдачи рассчитывается при помощи уравнения: V = U ∙ I ∙ sin ϕ.

Единицей измерения принят ватт. Возможно использование внесистемной меры подсчёта var, название которой составлено из английских слов volt, amper, reaction. Перевод на русский язык соответственно означает «вольт», «ампер», «обратное действие».

Если напряжение опережает ток, то смещение фаз считается больше нуля. В противном случае сдвиг фаз отрицательный. В зависимости от значения sin ϕ реактивная составляющая носит положительный или отрицательный характер. Присутствие в цепи индуктивной нагрузки позволяет говорить о реверсивной части больше нуля, а подключённый прибор потребляет энергию. Использование конденсаторов делает реактивную производительность минусовой, и устройство добавляет энергию в сеть.

Во избежание перегрузок и изменения установленного коэффициента мощности в цепи устанавливаются компенсаторы. Такие меры снижают потери электроэнергии, понижают искажения формы тока и позволяют использовать провода меньшего сечения.

В полную силу

Полная электрическая мощность определяет нагрузку, которую потребитель возлагает на сеть. Активная и реверсивная составляющие объединяются с полной мощностью уравнением: S = √ (P ² + V ²).

С индуктивной нагрузкой показатель V ˃ 0, а использование конденсаторов делает V ˂ 0. Отсутствие конденсаторов и катушек индуктивности делает реактивную часть равной нулю, что возвращает формулу к привычному виду: S = √ (P ² + V ²) = √ (P ² + 0) = √ P ² = P = U ∙ I. Полная мощность измеряется внесистемной единицей «вольт-ампер». Сокращённый вариант — В ∙ А.

Критерий полезности

Коэффициент мощности характеризует потребительскую нагрузку с точки зрения присутствия реактивной части работы. В физическом смысле параметр определяет сдвиг тока от приложенного напряжения и равен cos ϕ. На практике это означает количество тепла, выделяемого на соединительных проводниках. Уровень нагрева способен достигать существенных величин.

В энергетике коэффициент мощности обозначается греческой буквой λ. Диапазон изменения от нуля до единицы или от 0 до 100%. При λ = 1 подаваемая потребителю энергия расходуется на работу, реактивная составляющая отсутствует. Значения λ ≤ 0,5 признаются неудовлетворительными.

Безотказная работа приборов в электрической линии обусловлена правильным расчётом технических параметров. Найти мощность тока в цепи помогает набор формул, выведенных из законов Джоуля — Ленца и Ома. Принципиальная схема, грамотно составленная с учётом особенностей применяемых устройств, повышает производительность электросети.

Мощность постоянного тока

Мощность постоянного тока P – это величина, которая показывает какую работу совершил постоянный ток по перемещению электрического заряда за единицу времени.  Измеряется электрическая мощность, как и механическая – в ваттах.

Для того чтобы понять что такое электрическая мощность представим себе электрическое поле, в котором находится свободная частица.

Под действием напряженности E электрического поля, частица перемещается из точки a в точку b.  

При перемещении частицы из точки a в точку b электрическое поле совершает работу А. Эта работа зависит от напряженности, заряда и расстояния между a и b. 

Так как работа зависит еще и от величины заряда, то энергетической характеристикой электрического поля служит напряжение, которое является отношением работы A по перемещению заряда к величине самого заряда Q.

 

Если заряд равен единичному (Q=1), то получается, что напряжение это есть работа по перемещению единичного заряда из точки a в точку b.

 

Мощность определяется как отношение работы к  промежутку времени , за который была совершена эта работа.

 

Выходит, что мощность, затрачиваемая на единичный заряд равна

 

А на некоторое количество зарядов Q

 

Если присмотреться ко второму множителю, то можно рассмотреть в нем электрический ток, который выражен как скорость изменения заряда. Таким образом, получаем всем известную формулу

 

Для того чтобы узнать, какое количество энергии выделилось источником постоянного тока, нужно воспользоваться законом Джоуля –Ленца. 

Пример

Узнать какое количество энергии получит резистор от источника за 10 секунд, если его сопротивление равно 100 Ом, а ЭДС источника равно 12 В. Сопротивление источника принять равным нулю.

 

Найдем силу тока по закону Ома 

Посчитаем мощность

Такое количество энергии получает резистор за секунду, а за десять секунд он получит в десять раз больше

Рекомендуем прочесть статью о балансе мощностей и о мгновенной мощности.

  • Просмотров: 8825
  • формула для расчета по току и напряжению > Флэтора

    Формула для вычисления энергии электрических полей конденсаторов

    Определение и формулы напряженности электрополя. Работа и энергия в электростатическом поле. Электрическое поле в конденсаторе. Определение максимальной энергии в конденсаторах. Определение энергии электрического поля через составление формул для работы….

    28 04 2021 10:39:47

    Управление светодиодными лентами

    Знакомство с устройством светодиодных лент, способы регулирования их яркости и управление цветом. Подключение диммеров к светодиодным источникам света….

    16 04 2021 10:50:10

    Технические характеристики и расшифровка кабелей ВБбШв

    Маркировка установочных проводов и кабелей согласно Г О С Ту. Конструкция В Бб Шв: требования предъявляемые к изоляции провода. Технические характеристики В Бб Шв-провода. Конструктивные характеристики проводов В Бб- Шв (таблица).

    12 04 2021 16:28:38

    Диммер для паяльника своими руками

    Все кто занимается радиоэлектроникой, сталкивались с перегревом паяльника. Это может быть недорогой недавно купленный паяльник, который вышел из строя….

    01 04 2021 19:46:16

    Перечень средств относящихся к средствам индивидуальной защиты

    Определение средств индивидуальной защиты. Меры по снижению влияния вредных факторов, снижения степени опасности и предотвращения несчастных случаев. Перечень и классификация С И З. Порядок приобретения и выдачи, ответственность за использование….

    30 03 2021 17:22:30

    Проверка сопротивления резистора с помощью мультиметра

    Признаки повреждения резисторов. Проверка сопротивления мультиметром. Порядок проверки «подозрительного» резистора. Переменный резистор: правила проверки (прозвона). Измеряем позистор. Мультиметр: правила эксплуатации….

    27 03 2021 18:20:40

    Монтаж встраиваемых и выдвижных розетки

    Функционал места жительства сейчас на первом месте, именно поэтому стоит установить у себя выдвижные розетки их разновидности поражают воображение.

    24 03 2021 17:42:10

    Зарядное устройство для аккумулятора 18650

    Аккумуляторная батарея 18650: преимущества и недостатки, маркировка аккумулятора. Определение эффекта памяти аккумуляторных батарей. Порядок заряда А К Б-18650. Схемы зарядных устройств для аккумуляторов типа 18650….

    15 03 2021 8:43:17

    Как измерить пульсацию и ее коэффициенты для светового потока

    Определение и нормы коэффициентов пульсации светового потока. Причины и источники мерцаний. Измерение коэффициентов пульсаций световых потоков. Стробоскопический эффект: положительные стороны и негативные последствия. Способы борьбы с мерцаниями….

    11 03 2021 0:47:13

    В чем измеряются единицы емкости конденсаторов

    Единица измерения емкости в системе С И и других системах. Фарады через основные единицы системы. Определение кратных единиц ёмкости. Таблица перевода дольных единиц. Маркировка конденсаторов. Кодировка больших по размерам устройств…

    05 03 2021 7:38:49

    Технические характеристики и расшифровка КВВГНГ LS-кабелей

    Маркировка контрольных проводов и кабелей согласно Г О С Ту. Конструкция К В В Г Н Г LS: требования предъявляемые к изоляции провода. Технические характеристики К В В Г Н Г-провода. Конструктивные характеристики проводов К В- В Г Н Г (таблица)….

    07 02 2021 3:26:10

    Все о магнитных пускателях или контакторах серии ПМЛ

    История создания и назначение магнитного пускателя П М Л. Конструкция прибора и расшифровка цифробуквенного обозначения контакторов. Монтаж пускателей: крепление на DIN-рейке или крепление болтами. Подключение пускателя- П М Л….

    24 01 2021 20:55:32

    Индикатор короткозамкнутых витков своими руками: почему коротит

    Почему в проводах и контактах происходит короткое замыкание. Что такое короткозамкнутый виток. Причины и устранение коротких замыканий в кабелях и соединениях. В каких случаях коротит скрытая проводка. Короткие замыкания: как найти и внешние признаки….

    30 12 2020 22:39:53

    Закон Ома для неоднородного участка цепи

    Понятие и классическая формулировка закона Ома для неоднородного участка цепи. Что такое неоднородная цепь. Применение закона для неоднородных участков….

    22 12 2020 22:46:16

    Проверка стабилитрона на плате с помощью мультиметра

    Стабилитрон и его свойства. Проверка стабилитрона мультиметром на плате: порядок действий. Определение теплового пробоя. Проверка исправных стабилитронов. Пороговое значение напряжения. Можно ли проверить стабилитрон не выпаивая….

    08 12 2020 16:26:14

    Как считать электрическую мощность?

    Чтобы обеспечить нормальное функционирование электрической проводки, необходимо ещё на этапе проектирования правильно рассчитать мощность, подобрать кабель подходящего сечения. От этого зависит не только срок эксплуатации системы, но и пожаробезопасность сооружения. Если выбрать сечение ошибочно или неправильно рассчитать мощность, можно столкнуться с такими опасными последствиями, как возгорание электропроводки, короткие замыкания, пожар и пр. При выборе оборудования  и кабельно-проводниковой продукции важно учитывать разные критерии, среди которых напряжение, сила тока, особенности эксплуатации сети.

    Формула расчёта

    В уже функционирующей сети измерить мощность электрического тока можно при помощи специального оборудования. Что же делать на этапе проектирования? Ведь самой цепи ещё нет. В этом случае применяется расчётный метод.

    Существует два вида мощности: активная и реактивная. Активная превращается в полезную энергию безвозвратно, считается полезной. Реактивная предусматривает затрату определенного (расчетного согласно установленного оборудования и типа оборудования) количества энергии.

    В нашем случае реактивная мощность нам не интересна, и мы не будем ее рассчитывать!

    В цепях переменного тока, ток и напряжения сдвигаются относительно друг друга.

    Этот сдвиг на угол cos обозначается буквой φ (фи).

    При расчёте мощности электрической мощности следует учитывать тип сети:

    P=U*I*cosφ — для однофазной;

    P=√3*U*I*cosφ — для трехфазной.

    U – это напряжение сети,

     I – сила тока,

    cosφ – коэффициент мощности.

    cosφ – коэффициент мощности, это паспортная величина оборудования, если не известно о типе оборудования (например, квартиры), то cosφ – расчетный и берется из инструкции по проектированию (СП 256.1325800.2016)

    Зависимость коэффициента мощности

    Чтобы рассчитать полную (Обращаем внимание, что имеется ввиду установленная, т.е. полная мощность) мощность, необходимо определить суммарную мощность всей техники и оборудования, которые будут эксплуатироваться, и подключаться к данной электрической сети. Это можно узнать путём суммирования мощностей приборов (этот показатель указан в паспорте товара).

    При определении коэффициента мощности учитывается характер нагрузки. К примеру, для нагревательного оборудования он близится к 1. Важно учитывать, что любая активная нагрузка предполагает незначительную реактивную составляющую, поэтому коэффициент мощности будет равен не 1, а 0,95. Для более мощных приборов – 0,8. Напряжение для однофазных цепей принимается 220 В, для трехфазных – 380 В.

    Калькулятор расчета мощности онлайн | BBF.RU

    Мощность — ключевая характеристика электроприборов, показывающая, сколько энергии требуется для их работы. С понятием мощности мы знакомимся еще на уроках физики, и это знание необходимо нам в жизни как для решения простых вопросов о покупке подходящего зарядного устройства для смартфона, так и для профессиональных изысканий в области энергосбережения.

    Понятия тока, напряжения и работы

    Для упрощенного рассмотрения электрического тока и связанных с ним понятий напряжения, работы и мощности, возьмем простую аналогию с потоком жидкости. Представьте себе трубку, по которой течет вода. Жидкость может течь из-за разности высот разных точек гибкой трубки или под напором из крана. Поток воды обладает потенциальной энергией, которую можно использовать, направив его, например, на лопасти водяной мельницы. В этом случае вода начнет выполнять работу, приводя в движение жернова.

    Все тоже самое и с электрическим током. Если в проводнике в разных точках присутствует разный электрический потенциал, то в этом месте создается напряжение, заставляющее электроны перетекать от одной точки к другой. Это и есть электрический ток. Пока ток течет «впустую», он обладает некой энергией. Если мы направим ток на замкнутую металлическую рамку, находящуюся в магнитном поле, то рамка начнет вращаться, совершая работу, и именно по этому принципу работают электродвигатели.

    Работа воды на мельнице зависит от силы напора воды и ее объема. Сравните силу воды из водяного пистолета и пожарного брандспойта. Очевидно, что в последнем случае вода совершает гораздо большую работу. Аналогично все происходит и с электрическим током. Чем сильнее напряжение (напор) и сила тока (объем воды), тем большую работу мы можем выполнить. Естественно, любая работа выполняется не мгновенно, а в течение какого-то промежутка времени, даже если это миллисекунды. Математически для цепей постоянного тока это выражается следующей формулой:

    A = I × U × t,

    где I — сила тока, U — напряжение, t — время.

    Связь работы и мощности

    Мощность электрического тока оценивается по количеству работы, которая выполняется в течение заданного времени. Из курса физики мы знаем, что мощность постоянного тока выражается простой формулой:

    P = U × I

    Постоянный ток — это электрический ток, который не меняет своего направления. Если вернуться к аналогии выше, то напор воды — это именно постоянный ток. Переменный ток в замкнутой цепи с определенной частотой изменяет свою полярность, а это приводит к запаздыванию напряжения. Для цепей переменного тока требуется учитывать сдвиг напряжения, который в векторной алгебре косинусом угла fi между векторами тока и напряжения:

    P = U × I × cosfi

    Если же вектора сонаправлены (угол между векторами равен 0 градусов), то косинус 0 превращается в единицу. Очевидно, что в формуле работы мы можем заменить произведение U × I на мощность и получим простую формулу:

    A = P × t или P = A / t,

    из чего следует, что мощность — это количество работы, совершаемой за единицу времени.

    Наш калькулятор подходит для вычисления мощности постоянного или переменного тока: в формулу заложено итоговое значение мощности, поэтому нам не важен характер тока. Для использования инструмента достаточно заполнить две ячейки из трех, после чего неизвестное будет подсчитано автоматически. В школьных задачках вам потребуется вычислить мощность как произведение тока и напряжения, а в бытовых вопросах мощность всегда указывается на щитке электроприборов.

    Примеры из реальной жизни

    Школьная задача

    В простых задачах по физике не требуется промежуточных вычислений. Давайте попробуем вычислить время работы прибора, если его мощность составляет 300 Вт, а выполненная работа равна 65 000 Дж. Для решения нам достаточно заполнить соответствующие ячейки и получить ответ, что электроприбор работал 216 секунд.

    Бытовой расчет

    Давайте вычислим, на сколько времени непрерывной работы хватает обычного смартфона. Известно, что емкость аккумулятора составляет 4,1 А·ч, мощность смартфона равна 3 Вт, а напряжение заряда составляет 5 В. Мы знаем, что количество запасенной энергии можно подсчитать, просто умножив емкость на напряжение. Получим, что аккумулятор смартфона запасает 20,5 Вт·ч. Переведем Вт·ч в Дж, зная, что 1 Вт·ч = 3 600 Дж. Получим, что энергия, запасаемая аккумулятором смартфона, равна 73 800 Дж. Теперь у нас есть все данные. Введем в соответствующие ячейки значения энергии и мощности и получим, что телефон способен непрерывно работать 24 600 секунд или почти 8 часов. Похоже на правду.

    Заключение

    Мощность электроприборов напрямую связана со временем работы и запасаемой в аккумуляторах энергии. Используйте наш онлайн-калькулятор для простых вычислений времени работы, энергии или мощности, а также для решения простых задачек по физике.

    Мощность электрического тока: особенности и измерения

    Мощность электрического тока – скорость выполняемой цепью работы. Простое определение, морока с пониманием. Мощность подразделяется на активную, реактивную. И начинается…

    Работа электрического тока, мощность

    При движении заряда по проводнику поле выполняет над ним работу. Величина характеризуется напряжением, в отличие от напряженности в свободном пространстве. Заряды двигаются в сторону убывания потенциалов, для поддержания процесса требуется источник энергии. Напряжение численно равно работе поля при перемещении на участке единичного заряда (1 Кл). В ходе взаимодействий электрическая энергия переходит в другие виды. Поэтому необходим ввод универсальной единицы, физической свободно конвертируемой валюты. В организме мерой выступает АТФ, электричестве — работа поля.

    Электрическая дуга

    На схеме момент превращения энергии отображается в виде источников ЭДС. Если у генераторов направлены в одну сторону, у потребителя – обязательно в другую. Наглядным фактом отражается процесс расхода мощности, отбора у источников энергии. ЭДС несет обратный знак, часто называется противо-ЭДС. Избегайте путать понятие с явлением, возникающим в индуктивностях при выключении питания. Противо-ЭДС означает переход электрической энергии в химическую, механическую, световую.

    Потребитель хочет выполнить работу за некоторую единицу времени. Очевидно, газонокосильщик не намерен ждать зимы, надеется управиться к обеду. Мощность источника должна обеспечить заданную скорость выполнения. Работу осуществляет  электрический ток, следовательно, понятие также относится. Мощность бывает активной, реактивной, полезной и мощностью потерь. Участки, обозначаемые физическими схемами сопротивлениями, на практике вредны, являются издержками. На резисторах проводников выделяется тепло, эффект Джоуля-Ленца ведет к лишнему расходу мощности. Исключением назовем нагревательные приборы, где явление желательно.

    Полезная работа на физических схемах обозначается противо-ЭДС (обычный источник с обратным генератору направлением). Для мощности имеется несколько аналитических выражений. Иногда удобно использовать одно, в других случаях – иное (см. рис.):

    Выражения мощности тока

    1. Мощность – скорость выполнения работы.
    2. Мощность равна произведению напряжения на ток.
    3. Мощность, затрачиваемая на тепловое действие, равна произведению сопротивления на квадрат тока.
    4. Мощность, затрачиваемая на тепловое действие, равна отношению квадрата напряжения к сопротивлению.

    Запасшемуся токовыми клещами проще использовать вторую формулу. Вне зависимости от характера нагрузки посчитаем мощность. Только активную. Мощность определена многими факторами, включая температуру. Под номинальным для прибора значением понимаем, развиваемое в установившемся режиме. Для нагревателей следует применять третью, четвертую формулу. Мощность зависит целиком и полностью от параметров питающей сети. Предназначенные для работы со 110 вольт переменного тока в европейских условиях быстро сгорят.

    Трехфазные цепи

    Новичкам трехфазные цепи представляются сложными, на деле это более элегантное техническое решение. Даже электричество домом поставляют тремя линиями. Внутри подъезда делят по квартирам. Больше смущает то, что некоторые приборы на три фазы лишены заземления, нулевого провода. Схемы с изолированной нейтралью. Нулевой провод не нужен, ток возвращается источнику по фазным линиям. Разумеется, нагрузка здесь на каждую жилу повышенная. Требования ПУЭ отдельно оговаривают род сети. Для трехфазных схем вводятся следующие понятия, о которых нужно иметь представление, чтобы правильно посчитать мощность:

    Трехфазная цепь с изолированной нейтралью

    • Фазным напряжением, током называют, соответственно, разницу потенциалов и скорость передвижения заряда меж фазой и нейтралью. Понятно, в оговоренном выше случае с полной изоляцией формулы будут недействительны. Поскольку нейтрали нет.
    • Линейным напряжением, током называют, соответственно, разницу потенциалов или скорость перемещения заряда меж любыми двумя фазами. Номера понятны из контекста. Когда говорят о сетях 400 вольт, подразумевают три провода, разница потенциалов с нейтралью равна 230 вольт. Линейное напряжение выше фазного.

    Меж напряжением и током существует сдвиг фаз. О чем умалчивает школьная физика. Фазы совпадают, если нагрузка 100% активная (простые резисторы). Иначе появляется сдвиг. В индуктивности ток отстает от напряжения на 90 градусов, в емкости – опережает. Простая истина легко запоминается следующим образом (плавно подходим к реактивной мощности). Мнимая часть сопротивления индуктивности составляет jωL, где ω – круговая частота, равная обычной (в Гц), помноженной на 2 числа Пи; j – оператор, обозначающий направление вектора. Теперь пишем закон Ома: U = I R = I  jωL.

    Из равенства видно: напряжение нужно отложить вверх на 90 градусов при построении диаграммы, ток останется на оси абсцисс (горизонтальная ось Х). Вращение по правилам радиотехники происходит против часовой стрелки. Теперь очевиден факт: ток отстает на 90 градусов. По аналогии проведем сравнение для конденсатора. Сопротивление переменному току в мнимой форме выглядит так: -j/ωL, знак указывает: откладывать напряжение нужно будет вниз, перпендикулярно оси абсцисс. Следовательно, ток опережает по фазе на 90 градусов.

    В реальности параллельно с мнимой частью присутствует действительная – называют активным сопротивлением. Проволока катушки представлена резистором, будучи свитой, приобретает индуктивные свойства. Поэтому реальный угол фаз будет не 90 градусов, немного меньше.

    А теперь можно переходить к формулам мощности тока трехфазных цепей. Здесь линия формирует сдвиг фаз. Меж напряжением и током, и относительно другой линии. Согласитесь, без заботливо изложенных авторами знания факт нельзя осознать. Меж линиями промышленной трехфазной сети сдвиг 120 градусов (полный оборот – 360 градусов). Обеспечит равномерность вращения поля в двигателях, для рядовых потребителей безразличен. Так удобнее генераторам ГЭС – нагрузка сбалансированная. Сдвиг идет меж линиями, в каждой ток опережает напряжение или отстает:

    1. Если линия симметричная, сдвиги меж любыми фазами по току составляют 120 градусов, формула получается предельно простой. Но! Если нагрузка симметрична. Посмотрим изображение: фаза ф не 120 градусов, характеризует сдвиг меж напряжением и током каждой линии. Предполагается, включили двигатель с тремя равноценными обмотками, получается такой результат. Если нагрузка несимметрична, потрудитесь провести вычисления для каждой линии отдельно, затем сложить результаты воедино для получения общей мощности тока.
    2. Вторая группа формул приведена для трехфазных цепей с изолированной нейтралью. Предполагается, ток одной линии утекает по другой. Нейтраль отсутствует за ненадобностью. Поэтому напряжения берутся не фазные (не от чего отсчитывать), как предыдущей формулой, а линейные. Соответственно, цифры показывают, какой параметр следует взять. Повремените пугаться греческих букв – фазы меж двумя перемножаемыми параметрами. Цифры меняются местами (1,2 или 2,1), чтобы правильно учесть знак.
    3. В асимметричной цепи вновь появляются фазные напряжение, ток. Здесь расчет ведется отдельно для каждой линии. Никаких вариантов нет.

    Формулы мощности тока

    На практике измерить мощность тока

    Намекнули, можно воспользоваться токовыми клещами. Прибор позволит определить крейсерские параметры дрели. Разгон можно засечь только при многократных опытах, процесс чрезвычайно быстрый, частота смены индикации не выше 3-х раз в секунду. Токовые клещи демонстрируют погрешность. Практика показывает: достичь погрешности, указанной в паспорте, сложно.

    Чаще для оценки мощности используют счетчики (для выплат компаниям-поставщикам), ваттметры (для личных и рабочих целей). Стрелочный прибор содержит пару неподвижных катушек, по которым течет ток цепи, подвижную рамку, для заведения напряжения путем параллельного включения нагрузки. Конструкция рассчитана сразу реализовать формулу полной мощности (см. рис.). Ток умножается на напряжение и некий коэффициент, учитывающий градуировку шкалы, также на косинус сдвига фаз между параметрами. Как говорили выше, сдвиг умещается в пределах 90 – минус 90 градусов, следовательно, косинус положителен, крутящий момент стрелки направлен в одну сторону.

    Отсутствует возможность сказать индуктивная ли нагрузка или емкостная. Зато при неправильном включении в цепь показания будут отрицательными (завал набок). Произойдет аналогичное событие, если потребитель вдруг станет отдавать мощность обратно нагрузке (бывает такое). В современных приборах происходит нечто подобное же, вычисления ведет электронный модуль, интегрирующий расход энергии, либо считывающий показания мощности. Вместо стрелки присутствует электронный индикатор и множество других полезных опций.

    Особые проблемы вызывают измерения в асимметричных цепях с изолированной нейтралью, где нельзя прямо складывать мощности каждой линии. Ваттметры делятся принципом действия:

    1. Электродинамические. Описаны разделом. Состоят из одной подвижной, двух неподвижных катушек.
    2. Ферродинамические. Напоминает двигатель с расщепленным полюсом (shaded-pole motor).
    3. С квадратором. Используется амплитудно-частотная характеристика нелинейного элемента (например, диода), напоминающая параболу, для возведения электрической величины в квадрат (используется в вычислениях).
    4. С датчиком Холла. Если индукцию сделать при помощи катушки пропорциональной напряжению магнитного поля в сенсоре, подать ток, ЭДС будет результатом умножения двух величин. Искомая величина.
    5. Компараторы. Постепенно повышает опорный сигнал, пока не будет достигнуто равенство. Цифровые приборы достигают высокой точности.

    В цепях с сильным сдвигом фаз для оценки потерь применяется синусный ваттметр. Конструкция схожа с рассмотренной, пространственное положение таково, что вычисляется реактивная мощность (см. рис.). В этом случае произведение тока и напряжения домножим на синус угла сдвига фаз. Реактивную мощность измерим обычным (активным) ваттметром. Имеется несколько методик. Например, в трехфазной симметричной цепи нужно последовательную обмотку включить в одну линию, параллельную – в две другие. Затем производятся вычисления: показания прибора умножаются на корень из трех (с учетом, что на индикаторе произведение тока, напряжения и синуса угла между ними).

    Методика двух ваттметров

    Для трехфазной цепи с простой асимметрией задача усложняется. На рисунке показана методика двух ваттметров (ферродинамических или электродинамических). Начала обмоток указаны звездочками. Ток проходит через последовательные, напряжение с двух фаз подается на параллельную (одно через резистор). Алгебраическая сумма показаний обоих ваттметров складывается, умножается на корень из трех для получения значения реактивной мощности.

    Как рассчитать мощность электрического тока: формула для расчета по току и напряжению

    Пожаловалась бабушка соседка снизу: подарили мне дети моющий пылесос. Он прекрасно работает, но откуда-то идет запах гари.

    Пошел смотреть. Проводка у нас старая: лапша из алюминия 2,5 квадрата. А пылесос потребляет 2,5 kW. Прикинул, как работает формула расчета мощности по току и напряжению для этого случая.

    Разделил 2500 ватт на 220 вольт. Получил чуть больше 11 ампер. Наши провода держат нагрузку 22 А. Имеем практически двойной резерв по току. Другие потребители при уборке отключены.

    Стали проверять и нюхать: запах около квартирного щитка. Открыл, осмотрел: шина сборки ноля в саже, на одной перемычке горелая изоляция. Винт крепления ослаблен. Вот и причина начала возгорания. Исправил.

    На этом примере я показываю, что всегда надо оценивать мощность потребления электроприборов и возможности проводки с защитными устройствами. Об этом рассказываю ниже.

    Что такое мощность в электричестве: просто о сложном

    Вспомнилась былина об Илье Муромце, когда он приложил всю свою мощь к соловью разбойнику. У бедолаги сразу посыпались искры из глаз, как пламя с верхней картинки на проводке с неправильным монтажом.

    Простыми словами: мощность в электричестве — это силовая характеристика энергии, которой оценивают, как способности генераторных установок ее вырабатывать, так возможности потребителей и транспортных магистралей.

    Все эти участки должны быть точно смонтированы и налажены для обеспечения безопасной работы. Как только в любом месте возникает неисправность, так сразу развивается авария во всей схеме.

    Если говорить о домашнем электрическом оборудовании, то приходится постоянно соблюдать баланс между:

    1. включенными в сеть приборами;
    2. конструкцией проводов и кабелей;
    3. настройкой защитных устройств.

    Только комплексное решение этих трех вопросов может обеспечить безопасность проводки и жильцов.

    Как рассчитать электрическую мощность в быту

    Формулы расчета мощности в электричестве позволяют выполнить качественную оценку безопасности каждого из перечисленных выше пунктов.

    Пользоваться ими не сложно. Я уже приводил в предыдущих статьях шпаргалку электрика, где они помещены в наглядной форме для цепей постоянного тока.

    Они полностью справедливы для активной составляющей мощности переменного тока, совершающей полезную работу. Кстати, кроме нее есть еще и бесполезная — реактивная, связанная с потерями энергии. Ее описанию посвящен второй раздел.

    Такие вычисления удобно делать с помощью онлайн калькулятора. Он избавляет от рутинных математических вычислений и арифметических ошибок.

    При любом из способов для расчета активной мощности требуется знать две из трех электрических величин:

    1. силу тока I;
    2. приложенное напряжение U;
    3. сопротивление участка цепи R.

    Как измерить электрическую мощность дома

    Существует еще одна возможность оценки активной мощности: ее измерение в действующей схеме специальными приборами: ваттметрами.

    Точные замеры может обеспечить промышленный лабораторный ваттметер. Он изготавливается как прибор, работающий на аналоговых сигналах,так и с помощью цифровых технологий.

    В бытовой проводке точные вычисления не нужны. Для нее выпускаются различные виды более простых ваттметров.

    Популярностью пользуются приборы, которые можно вставить в розетку и подключить к ним шнур питания от потребителя, включить их в работу и сразу снять показания на дисплее в ваттах.

    Их так и называют: ваттметр розетка. Они измеряют чисто активную мощность переменного тока.

    Такие приборы избавляют электрика от выполнения сложных операций под напряжением, когда требуется замерять:

    • действующее напряжение;
    • силу тока;
    • угол сдвига фаз между векторами тока и напряжения.

    Потом все данные дополнительно требуется вводить в формулу расчета мощности по току и напряжению, делать по ней вычисления.

    Этот метод можно упростить, если внимательно наблюдать за показаниями электрического счетчика индукционной системы с вращающимся диском. Он считает совершенную работу: потребленную мощность за определенную время.

    Однако скорость вращения диска как раз и характеризует величину потребления. Надо просто посчитать сколько раз он обернется за минуту и перевести в ватты по табличке, расположенной на корпусе.

    Почему реактивное сопротивление схемы влияет на мощность переменного тока

    Синусоидальная гармоника напряжения, поступая на резистивное сопротивление, изменяет величину тока без его отклонения на комплексной плоскости.

    Такой ток совершает полезную работу с минимальными потерями энергии, вырабатывая активную мощность. Частота колебания сигнала не оказывает на нее никакого влияния.

    Сопротивление конденсатора и индуктивности зависит от частоты гармоники. Его противодействие отклоняет направление тока на каждом из этих элементов в разные стороны.

    Такие процессы связаны с потерей части энергии на бесполезные преобразования. На них расходуется мощность Q, которую называют реактивной.Ее влияние на полную мощность S и связь с активной P удобно представлять графически прямоугольным треугольником.

    Захотелось его нарисовать на фоне оборудования из нагромождений фарфора и металла, где пришлось поработать довольно долго.Отвлекся. Не судите за это строго.

    Сравните его с опубликованным мною ранее треугольником сопротивлений. Находите общие черты?

    Ими являются геометрические пропорции фигуры, описывающие их формулы и угол φ, определяющий потери полной мощности. Перехожу к их более подробному рассмотрению.

    Формулы расчета мощности для однофазной и трехфазной схемы питания

    В идеальном теоретическом случае трехфазная схема состоит из трех одинаковых однофазных цепей. На практике всегда есть какие-то отклонения. Но, в большинстве случаев при анализах ими пренебрегают.

    Поэтому рассматриваем вначале наиболее простой вопрос.

    Графики и формулы под однофазное напряжение

    Как работает резистор

    На чисто резистивном сопротивлении синусоиды тока и напряжения совпадают по углу, направлены на каждом полупериоде одинаково.Поэтому их произведение, выражающее мощность, всегда положительно.

    Его значение в произвольный момент времени t называют мгновенным, обозначая строчной буквой p.

    Среднее значение мощности в течение одного периода называют активной составляющей. Ее график для переменного тока имеет фигуру симметричного всплеска с максимальным значением Pm в середине каждого полупериода Т/2.

    Если взять половину его величины Pm/2 и провести прямую линию в течении одного периода Т, то получим прямоугольник с ординатой P.

    Его площадь равна двум площадям графиков активной составляющих одного любого полупериода. Если посмотреть на картинку внимательнее, то можно представить, что верхняя часть всплеска отрезана,перевернута и заполнила свободное пространство внизу.

    Представление этого графика помогает запомнить, что на активном сопротивлении мощность постоянного и переменного тока вычисляется по одной формуле, не меняет своего знака.

    График мгновенных значений активной мощности переменного тока на резистивном сопротивлении имеет вид повторяющихся положительных волн. Но за один период им совершается такая же работа, как и в цепях постоянного тока и напряжения.

    На резисторе не создается реактивных потерь.

    Как работает индуктивность

    Катушка с обмоткой своими витками запасает энергию магнитного поля. Благодаря процессу ее накопления индуктивное сопротивление отодвигает вперед на 90 градусов вектор тока относительно приложенного напряжения на комплексной плоскости.

    Перемножая их мгновенные величины получаем значения мощности, которое за один период меняет знаки (направление) в каждом полупериоде.

    Частота изменения мощности на индуктивности в два раза выше,чем у ее составляющих: синусоид тока и напряжения. Она состоит из двух частей:

    1. активной, обозначаемой индексом PL;
    2. реактивной QL.

    Реактивная часть на индуктивности создается за счет постоянного обмена энергией между катушкой и приложенным источником. На ее величину влияет значение индуктивного сопротивления XL.

    Как работает конденсатор

    Емкость конденсатора постоянно накапливает заряд между своими обкладками. За счет этого происходит сдвиг вектора тока вперед на 90 градусов относительно приложенного напряжения.

    График мгновенной мощности напоминает вид предыдущего, но начинается с отрицательной полуволны.

    Реактивная составляющая, выделяемая на конденсаторе, зависит от величины емкостного сопротивления XC.

    Как работает реальная схема со всеми видами сопротивлений

    В чистом виде приведенные выше графики и выражения встречаются не так часто. На самом деле передача электроэнергии и ее работа на переменном токе связаны с комплексным преодолением сил электрического сопротивления резисторов, конденсаторов и индуктивностей.

    Причем, какая-то из этих составляющих будет преобладать. Для таких случаев преобразования электрической энергии в мгновенную мощность могут иметь один из следующих видов.

    На верхней картинке показан случай, когда вектор тока отстает от приложенного напряжения, а на нижней — опережает.

    В обоих случаях величина активной составляющей уменьшается от значения полной на значение, выражаемое как cosφ. Поэтому его принято называть коэффициентом мощности.

    Косинус фи (cosφ) используется при анализе треугольника мощностей и сопротивлений, характеризует потери энергии.

    Как работает схема трехфазного электроснабжения

    На ввод распределительного щита многоэтажного здания поступает трехфазное напряжение от электроснабжающей организации, вырабатываемое промышленными генераторами.

    Его же, за отдельную плату, при желании может подключить владелец частного дома, что многие и делают. При этом рабочая схема и диаграмма напряжений выглядит следующим образом.

    В старой системе заземления TN-C она выполняется четырехпроводным подключением, а у новой TN-S — пятипроводным с добавлением защитного РЕ проводника. Его на этой схеме я не показываю для упрощения.

    Каждую из фаз при работе необходимо стараться нагружать одинаково равными по величине токами. Тогда в домашней проводке будет создаваться наиболее благоприятный оптимальный режим без опасных перекосов энергии.

    • В этом случае формула расчета мощности по току и напряжению для трехфазной схемы может быть представлена простой суммой аналогичных формул для составляющих однофазных цепей.
    • А поскольку они все идентичные, то их просто утраивают.
    • Например, когда активная мощность фазы В имеет выражением Рв=Uв×Iв×cosφ, то для всей трехфазной схемы она будет выражена следующей формулой:

    Р = Рa+Рв+Рc

    Если пометить фазное выражение буквой ф. например Pф, томожно записать:

    P = 3Pф = 3Uф×Iф×cosφ

    Аналогично будет вычисляться реактивная составляющая

    Q = Qa+Qв+Qc

    Или

    Q = 3Qф = 3Uф×Iф×sinφ

    Поскольку P и Q представляют величины катетов прямоугольного треугольника, то гипотенузу или полную составляющую можно вычислить как квадратный корень из суммы их квадратов.

    S = √(P2+Q2)

    Как учитывается трехфазная полная мощность

    В энергосистеме, да и в частном доме, требуется анализировать подключенные нагрузки, равномерно распределять их по источникам напряжений.

    С этой целью работают многочисленные конструкции измерительных приборов. На щитах управления подстанций расположены щитовые ваттметры и варметры, предназначенные для работы в разных долях кратности.

    Старые аналоговые приборы показаны на этой картинке.

    Для того, чтобы не путаться в записях вычислений введены разные наименования единиц. Они обозначаются:

    • ВА — (русское), VA (международное) вольтампер для полной величины мощности;
    • Вт —(русское), var (международное) ватт —активной;
    • вар (русское), var (международное) — реактивной.

    Аналоговые приборы измеряют только активную или реактивную составляющую, а полную величину необходимо вычислять по формулам.

    Многие современные цифровые приборы способны осуществлять эту функцию автоматически.

    Видеоурок Павла Виктор дополняет мой материал. Рекомендую посмотреть.

    Калькулятор мощности для своих

    Здесь вы можете выполнить вычисления онлайн без использования формул и арифметических действий. Просто введите ваши исходные данные в таблицу и жмите кнопку “Рассчитать ток”.

    А в заключение напоминаю, что для ваших вопросов создан раздел комментариев. Задавайте их, я отвечу.

    Источник: https://ElectrikBlog.ru/formula_rascheta_moshchnosti_po_toku_i_napryazheniyu_elektroskhemy/

    Как рассчитать мощность по току и напряжению?

    Любой из элементов электрической сети является материальным объектом определенной конструкции. Но его особенность состоит в двойственном состоянии. Он может быть как под электрической нагрузкой, так и обесточен.

    Если электрического подключения нет, целостности объекта ничто не угрожает.

    Но при присоединении к источнику электропитания, то есть при появлении напряжения (U) и электротока, неправильная конструкция элемента электросети может стать для него фатальной, если напряжение и электроток приведут к выделению тепла.

    Далее из статьи наши читатели получат информацию о том, как правильно сделать расчет мощности по току и напряжению, чтобы электрические цепи работали исправно и продолжительно.

    Отличия мощности при постоянном и переменном напряжении

    Наиболее простым получается расчет мощности электрических цепей на постоянном электротоке. Для их участков справедлив закон Ома, в котором задействовано только приложенное U, и сопротивление. Чтобы рассчитать силу тока I, U делится на сопротивление R:

    • I=U/R ,
    • причем искомая сила тока именуется амперами.
    • А поскольку электрическая мощность Р для такого случая — это произведение U и силы электротока, она так же легко, как и электроток, вычисляется по формуле:
    • P=U*I ,
    • причем искомая мощность нагрузки именуется ваттами.

    Все компоненты этих двух формул характерны для постоянного электротока и называются активными. Напоминаем нашим читателям, что закон Ома, позволяющий выполнить расчет силы тока, весьма многообразен по своему отображению.

    Его формулы учитывают особенности физических процессов, соответствующих природе электричества. А при постоянном и переменном U они протекают существенно отличаясь. Трансформатор на постоянном U — это абсолютно бесполезное устройство.

    Также как синхронные и асинхронные движки.

    Принцип их функционирования заключен в изменяющемся магнитном поле, создаваемом элементами электрических цепей, обладающими индуктивностью. А такое поле появляется только как следствие переменного U и соответствующего ему переменного тока.

    Но электричеству свойственно также и накопление зарядов в элементах электрических цепей. Это явление называется электрической емкостью и лежит в основе конструкции конденсаторов.

    Параметры, связанные с индуктивностью и емкостью, называют реактивными.

    Расчет мощности в цепях переменного электротока

    Поэтому, чтобы определить ток по мощности и напряжению как в обычной электросети 220 В, так и в любой другой, где используется переменное U, потребуется учесть несколько активных и реактивных параметров.

    Для этого применяется векторное исчисление. В результате отображение рассчитываемой мощности и U имеет вид треугольника. Две стороны его — это активная и реактивная составляющие, а третья — их сумма.

    Например, полная мощность нагрузки S, именуемая вольт-амперами.

    Реактивная составляющая называется варами. Зная величины сторон для треугольников мощности и U, можно выполнить расчет тока по мощности и напряжению. Как это сделать, поясняет изображение двух треугольников, показанное далее.


    Треугольники мощности и напряжения

    Для измерения мощности применяются специальные приборы. Причем их многофункциональных моделей совсем мало.

    Это связано с тем, что для постоянного электротока, а также в зависимости от частоты используется соответствующий конструктивный принцип измерителя мощности.

    По этой причине прибор, предназначенный для измерения мощности в цепях переменного электротока промышленной частоты, на постоянном электротоке или на повышенной частоте будет показывать результат с неприемлемой погрешностью.


    Лабораторный ваттметр


    Щитовой ваттметр

    У большинства наших читателей выполнение того или иного вычисления с использованием величины мощности скорее всего происходит не с измеренным значением, а по паспортным данным соответствующего электроприбора.

    При этом можно легко рассчитать ток для определения, например, параметров электропроводки или соединительного шнура. Если U известно, а оно в основном соответствует параметрам электросети, расчет тока по мощности сводится к получению частного от деления мощности и U.

    Полученный таким способом расчетный ток определит сечение проводов и тепловые процессы в электрической цепи с электроприбором.  

    Но вполне закономерен вопрос, как рассчитать ток нагрузки при отсутствии каких-либо сведений о ней? Ответ следующий. Правильный и полный расчет тока нагрузки, запитанной переменным U, возможен на основании измеренных данных.

    Они должны быть получены с применением прибора, который замеряет фазовый сдвиг между U и электротоком в цепи. Это фазометр. Полный расчет мощности тока даст активную и реактивную составляющие.

    Они обусловлены углом φ, который показан выше на изображениях треугольников.


    Лабораторный фазометр


    Щитовой фазометр

    Используем формулы

    Этот угол и характеризует фазовый сдвиг в цепях переменного U, содержащих индуктивные и емкостные элементы. Чтобы рассчитывать активные и реактивные составляющие, используются тригонометрические функции, применяющиеся в формулах. Перед тем как посчитать результат по этим формулам, надо, используя калькуляторы или таблицы Брадиса, определить sin φ и cos φ. После этого по формулам

    я вычислю искомый параметр электрической цепи.

    Но следует учесть то, что каждый из параметров, рассчитанный по этим формулам, из-за U, постоянно изменяющегося по законам гармонических колебаний, может принимать либо мгновенное, либо среднеквадратичное, либо промежуточное значение.

    Три формулы, показанные выше, справедливы при среднеквадратичных значениях силы электротока и U. Каждое из двух остальных значений является результатом расчетной процедуры с использованием другой формулы, учитывающей ход времени t:

    Но и это еще не все нюансы. Например, для линий электропередачи применяются формулы, в которых фигурируют волновые процессы. И выглядят они по-другому. Но это уже совсем другая история…  

    Источник: https://domelectrik.ru/elektrosnabzhenie/bezopasnost/raschet-moshchnosti

    Формула мощности электрического тока, расчет по мощности и напряжению

    Для того, чтобы обеспечить безопасность при эксплуатации промышленных и бытовых электрических приборов, необходимо правильно вычислить сечение питающей проводки и кабеля. Ошибочный выбор сечения жил кабеля может привести из-за короткого замыкания к возгоранию проводки и к возникновению пожара в здании.

    Что такое мощность (Р) электротока

    Электрическая мощность является физической величиной, характеризующей скорость преобразования или передачи электрической энергии. Единицей измерения по Международной системе единиц (СИ) является ватт, в нашей стране обозначается Вт, международное обозначение — W.

    Что влияет на мощность тока

    На мощность (Р) влияет величина силы тока и величина приложенного напряжения. Расчет параметров электроэнергии выполняется еще на стадии проектирования электрических сетей объекта.

    Полученные данные позволяют правильно выбрать питающий кабель, к которому будут подключаться потребители. Для расчетов силы электротока используется значения напряжения сети и полной нагрузки электрических приборов.

    В соответствии с величиной силы электротока выбирается сечение жил кабелей и проводов.

    Отличия мощности при постоянном и переменном напряжении

    Ведем обозначения электрических величин, которые приняты в нашей стране:

    • Р − активная мощность, измеряется в ваттах, обозначается Вт;
    • Q − реактивная мощность, измеряется в вольт амперах реактивных, обозначается ВАр;
    • S − полная мощность, измеряется в вольт амперах, обозначается ВА;
    • U − напряжение, измеряется в вольтах, обозначается ВА;
    • I − ток, измеряется в амперах, обозначается А;
    • R − сопротивление, измеряется в омах, обозначается Ом.

    Назовем основные отличия P на постоянном и Q на переменном электротоке. Расчет P на постоянном электротоке получается наиболее простым. Для участков электрической цепи справедлив закон Ома. В этом законе задействованы только величина приложенного U (напряжения) и величина сопротивления R.

    Расчет S (полной мощности) на переменном электротоке производится несколько сложнее. Кроме P, имеется Q и вводится понятие коэффициента мощности. Алгебраически складывая активную P и реактивную Q, получают общую S.

    По какой формуле вычисляется

    Расчет силы тока по мощности и напряжению в сети постоянного тока

    Для расчета силы I (тока), надо величину U (напряжения) разделить на величину сопротивления.

    • Расчет силы тока по мощности и напряжению:
    • I = U ÷ R
    • Измеряется в амперах.

    Для такого случая электрическую Р (активную мощность) можно посчитать как произведение силы электрического I на величину U.

    Формула расчета мощности по току и напряжению:

    P = U × I

    Все компоненты в этих двух формулах характерны для постоянного электротока и их называют активными.

    1. Исходя из этих двух формул, можно вывести также еще две формулы, по которым можно узнавать P:
    2. P = I2 × R
    3. P = U2 ÷ R

    Однофазные нагрузки

    В однофазных сетях переменного электротока требуется произвести вычисление отдельно для Р и Q нагрузки, затем надо при помощи векторного исчисления их сложить.

    В скалярном виде это будет выглядеть так:

    S = √P2 + Q2

    В результате расчет P, Q, S имеет вид прямоугольного треугольника. Два катета этого треугольника представляют собой P и Q составляющие, а гипотенуза — их алгебраическую сумму.

    S измеряется в вольт-амперах (ВА), Q измеряется в вольт-амперах-реактивных (ВАр), Р измеряется в ваттах (Вт).

    Зная величины катетов для треугольников, можно рассчитать коэффициент мощности (cos φ). Как это сделать, показано на изображении треугольника.

    Расчет в трехфазной сети

    Переменный I (ток) отличается от постоянного по всем параметрам, особенно наличием нескольких фаз. Расчет P в трехфазной нагрузке необходим для правильного определения характеристик подключаемой нагрузки. Трехфазные сети широко применяются в связи с удобством эксплуатации и малыми материальными затратами.

    Трехфазные цепи могут соединяться двумя способами – звездой и треугольником. На всех схемах фазы обозначают символами А, В, С. Нейтральный провод обозначают символом N.

    При соединении звездой различают два вида U (напряжения) – фазное и линейное. Фазное U определяется как U между фазой и нейтральным проводом. Линейное U определяется как U между двумя фазами.

    Эти два U связаны между собой соотношением:

    UЛ = UФ × √3

    Линейные и фазные электротоки при соединении звездой равны друг другу: IЛ = IФ

    Форма расчета S при соединении звездой:

    S = SA + SB + SC = 3 × U × I

    Активная P:

    Р = 3 × Uф × Iф × cosφ

    Реактивная Q:

    Q = √3 × Uф × Iф × sinφ.

    При соединении треугольником фазное и линейное U равны друг другу: UЛ = UФ

    Линейный I при соединении треугольником определяется по формуле:

    IЛ = IФ × √3

    Формулы мощности электрического тока при соединении треугольником:

    • S = 3 × Sф = √3 × Uф × Iф;
    • Р = √3 × Uф × Iф × cosφ;
    • Q = √3 × Uф × Iф × sinφ.

    Средняя P в активной нагрузке

    В электрических сетях P измеряют при помощи специального прибора – ваттметра. Схемы подключения находятся в зависимости от способа подключения нагрузки.

    При симметричной нагрузке P измеряется в одной фазе, а полученный результат умножают на три. В случае несимметричной нагрузки для измерения потребуется три прибора.

    Параметры P электросети или установки являются важными данными электрического прибора. Данные по потреблению P активного типа передаются за определенный период времени, то есть передается средняя потребляемая P за расчетный период времени.

    Подбор номинала автоматического выключателя

    Автоматические выключатели защищают электрические аппараты от токов короткого замыкания и перегрузок.

    При аварийном режиме они обесточивают защищаемую цепь при помощи теплового или электромагнитного механизма расцепления.

    Тепловой расцепитель состоит из биметаллической пластины с различными коэффициентами теплового расширения. Если номинальный ток превышен, пластина изгибается и приводит в действие механизм расцепления.

    У электромагнитного расцепителя имеется соленоид с подвижным сердечником. При превышении заданного I, в катушке увеличивается электромагнитное поле, сердечник втягивается в катушку соленоида, в результате чего срабатывает механизм расцепления.

    • Минимальный I, при котором тепловой расцепитель должен сработать, устанавливается с помощью регулировочного винта.
    • Ток срабатывания у электромагнитного расцепителя при коротком замыкании равен произведению установленного срабатывания на номинальный электроток расцепителя.

    Видео о законах электротехники

    Из следующего видео можно узнать, что такое электричество, мощность электрического тока. Даны примеры практического применения законов электротехники.

    Источник: https://vdome.club/materialy/raschety/formula-moschnosti.html

    Мощность ток напряжение. Расчёт нагрузки и выбор питающих кабелей

    Электроэнергия давно используется человеком для удовлетворения своих потребностей, но она невидима, не воспринимается органами чувств, потому сложна для понимания. Мощность ток напряжение, все эти характеристики электроэнергии исследованы известными учеными, которые дали им определения и описали математическими методами взаимные связи между ними.

       Мощность ток напряжение сопротивление 

    Так же следует помнить, на величину электрического сопротивления влияет несколько факторов:

    • строение вещества, определяющее наличие свободных электронов в проводнике и влияющее на удельное сопротивление
    • площадь поперечного сечения и длина токовода
    • температура

    В приведенной таблице показаны общие соотношения для цепей постоянного и переменного тока, которые можно применять для анализа работы схем электроснабжения. 

    Расчёт сечения питающего кабеля и проводки

    Для обеспечения безопасности при эксплуатации бытовых электроприборов необходимо верно вычислить сечение питающего кабеля и проводки. Поскольку ошибочно выбранное сечение жил кабеля способно привести к перегреву провода, плавление его изоляции и в итоге, возгоранию, из-за короткого замыкания. 

       Мощность ток напряжение, удобная шпаргалка

    Основным параметром, по которому производят расчет сечения провода, является его продолжительная допустимая токовая нагрузка. Т.

    е, это такая номинальная величина тока, которую проводник способен через себя пропускать на протяжении длительного времени.

    Для определения величины номинального тока, необходимо знать приблизительную мощность всех подключаемых электроприборов и оборудования в квартире.

    И так, что мы имеем:

    • От значения величины тока зависит выбор питающего кабеля (провода), по которому могут быть подключены приборы энергопотребления к сети
    • Зная напряжение электрической сети и полную нагрузку электроприборов, можно по формуле вычислить силу тока, который потребуется пропускать по проводнику(проводу, кабелю). По его величине выбирают площадь сечения жил.

    Расчет тока, выполняем самостоятельно

    Если известны электро-потребители в квартире или доме, необходимо выполнить несложные расчёты, чтобы правильно смонтировать схему электроснабжения.

    Аналогичные расчёты выполняются для производственных целей: определения необходимой площади сечения жил кабеля при осуществлении подключения промышленного оборудования (различных промышленных электрических двигателей и механизмов).

    Мощность ток напряжение, расчёты для однофазной сети 220 В

    Сила тока I (в амперах, А) подсчитывается по формуле:

    • I = P / U,
    • где
    • P – электрическая полная нагрузка (обязательно указывается в техническом паспорте устройства), Вт (ватт)
    • U – напряжение электрической сети, В (вольт)

    Ниже в таблице представлены величины нагрузки типичных бытовых электроприборов и потребляемый ими ток (для напряжения 220 В).

    ЭлектроприборПотребляемая мощность, ВтСила тока, А
    Стиральная машина2000 – 25009,0 – 11,4
    Джакузи2000 – 25009,0 – 11,4
    Электроподогрев пола800 – 14003,6 – 6,4
    Стационарная электрическая плита4500 – 850020,5 – 38,6
    СВЧ печь900 – 13004,1 – 5,9
    Посудомоечная машина2000 — 25009,0 – 11,4
    Морозильники, холодильники140 — 3000,6 – 1,4
    Мясорубка с электроприводом1100 — 12005,0 — 5,5
    Электрочайник1850 – 20008,4 – 9,0
    Электрическая кофеварка6з0 — 12003,0 – 5,5
    Соковыжималка240 — 3601,1 – 1,6
    Тостер640 — 11002,9 — 5,0
    Миксер250 — 4001,1 – 1,8
    Фен400 — 16001,8 – 7,3
    Утюг900 — 17004,1 – 7,7
    Пылесос680 — 14003,1 – 6,4
    Вентилятор250 — 4001,0 – 1,8
    Телевизор125 — 1800,6 – 0,8
    Радиоаппаратура70 — 1000,3 – 0,5
    Приборы освещения20 — 1000,1 – 0,4

    Различные потребители электроэнергии подключаются через соответствующие автоматы к электросчётчику и далее общему автомату, который должен быть рассчитан на нагрузку приборов, которыми будет оборудована квартира. Провод, который подводит питание также должен удовлетворять нагрузке энергопотребителей.

    Как рассчитать ток защитного автомата

    Для группы розеток, предназначенных для питания бытовых электроприборов на кухне, необходимо подобрать защитный автоматический выключатель. Мощности приборов по паспортным данным составляют 2,0, 1,5 и 0,6 кВт.

    Решение. В квартире используется однофазная переменная сеть 220 вольт. Общая мощность всех приборов, подключенных в работу одновременно, составит 2,0+1,5+0,6=4,1 кВт=4100 Вт.

    По формуле I = P / U определим общий ток группы потребителей: 4100/220=18,64 А.

    Ближайший по номиналу автоматический выключатель имеет величину срабатывания 20 ампер. Его и выбираем. Автомат меньшего значения на 16 А будет постоянно отключаться от перегрузки.

    Ниже приводится таблица для скрытой проводки при однофазной схеме подключения квартиры для подбора провода при напряжении 220 В

    Сечение жилы провода, мм2Диаметр жилы проводника, ммМедные жилыАлюминиевые жилы
    Ток, АМощность, ВтТок, АМощность, кВт
    0,500,8061300
    0,750,98102200
    1,001,13143100
    1,501,38153300102200
    2,001,60194200143100
    2,501,78214600163500
    4,002,26275900214600
    6,002,76347500265700
    10,003,575011000388400
    16,004,5180176005512100
    25,005,64100220006514300

    Как видно из таблицы сечение жил зависит кроме нагрузки и от материала, из которого изготовлен провод.

    Мощность ток напряжение, расчёты для трёхфазной сети 380 В

    При трёхфазном электроснабжении сила тока I (в амперах, А) вычисляется по формуле:

    1. I = P /1,73 U,
    2. где P -потребляемая мощность, Вт;
    3. U — напряжение в сети, В,
    4. так как напряжение при трёхфазной схеме электроснабжения 380 В, формула примет вид:
    5. I = P /657, 4.

    Сечение жил в питающем кабеле при различной нагрузке при трёхфазной схеме напряжением 380 В для скрытой проводки представлена в таблице.

    Сечение жилы провода, мм2Диаметр жилы проводника, ммМедные жилыАлюминиевые жилы
    Ток, АМощность, ВтТок, АМощность, кВт
    0,500,8062250
    0,750,98103800
    1,001,13145300
    1,501,38155700103800
    2,001,60197200145300
    2,501,78217900166000
    4,002,262710000217900
    6,002,763412000269800
    10,003,5750190003814000
    16,004,5180300005520000
    25,005,64100380006524000

    Для расчёта тока в цепях питания нагрузки, характеризующейся большой реактивной полной мощностью, что характерно применению электроснабжения в промышленности:

    • электрические двигатели
    • дроссели приборов освещения
    • сварочные трансформаторы
    • индукционные печи

    В мощных приборах и оборудовании, доля реактивной нагрузки выше и поэтому для таких приборов в расчетах коэффициент мощности принимают равным 0,8.

    На практике принято считать, что при подсчёте электрических нагрузок для бытовых целей запас мощности принимают 5%. В случае расчёта электрических сетей для промышленного производства запас мощности принимают 20%.

    Будем рады, если подпишетесь на наш Блог!

    [wysija_form id=»1″]
    • ТЕГИ
    • Монтаж кабеля
    • Предварительный монтаж
    • Схемы

    Источник: https://powercoup.by/stati-po-elektromontazhu/moshhnost-tok-napryazhenie

    Расчет мощности по току и напряжению, схема и таблицы

    Главная страница » Ремонт квартиры » Электрика » Расчет мощности по току и напряжению, схема и таблицы.

    Расскажите друзьям и коллегам в социальных сетях!

    Чтобы обезопасить себя при работе с бытовыми электроприборами, необходимо в первую очередь правильно вычислить сечение кабеля и проводки. Потому-что если будет неправильно выбран кабель, это может привести к короткому замыканию, из за чего может произойти возгорание в здание, последствия могут быть катастрофическими.

    Это правило относиться и к выбору кабеля для электродвигателей.

    Расчёт мощности по току и напряжению

    Данный расчет происходит по факту мощности, проделывать его необходимо еще до начала проектирование своего жилища (дома, квартиры).

    • Из этого значение  зависят кабеля питающие приборы которые подключены к электросети.
    • По формуле можно вычислить силу тока, для этого понадобиться взять точное напряжение сети и нагрузку питающихся приборов. Ее величина дает нам понять площадь сечение жил.

    Если вам известны все электроприборы, которые в будущем должны питаться от сети, тогда можно легко сделать расчеты для схемы электроснабжение. Эти же расчеты можно выполнять и для производственных целей.

    Однофазная сеть напряжением 220 вольт
    • Формула силы тока I (A — амперы):
    • I=P/U
    • Где P — это электрическая полная нагрузка (ее обозначение обязательно указывается в техническом паспорте данного устройства), Вт — ватт;
    • U — напряжение электросети, В (вольт).
    • В таблице представлены стандартные нагрузки электроприборов и потребляемый ими ток (220 В).
    • На рисунке вы можете видет схему устройства электроснабжение дома при однофазном подключении к сети 220 вольт.

    Схема приборов при однофазном напряжении

    Как и показано на рисунке, все потребители должны быть подключены к соответствующим автоматам и счетчику, далее к общему автомату который будет выдерживать общею нагрузку дома. Кабель который будет доводит ток, должен выдерживать нагрузку всех подключенных бытовых приборов.

    В таблице ниже показана скрытая проводка при однофазной схеме подключение жилища для подбора кабеля при напряжении 220 вольт.

    Как и показано в таблице, сечение жил зависит и от материала из которого изготовлен.

    Трёхфазная сеть напряжением 380 В
    1. В трехфазном электроснабжении сила тока рассчитывается по следующей формуле:
    2. I = P /1,73 U
    3. P — потребляемая мощность в ватах;
    4. U — напряжение сети в вольтах.
    5. В техфазной схеме элетропитания 380 В, формула имеет следующий вид:
    6. I = P /657, 4
    7. Если к дому будет проводиться трехфазная сеть 380 В, то схема подключения будет иметь следующий вид.
    8. В таблице ниже представлена схема сечения жил в питающем кабеле при различной нагрузке при трехфазном напряжении 380 В для скрытой проводки.
    9. Для дальнейшего расчета питания в цепях нагрузки, характеризующейся большой реактивной полной мощностью, что характерно применению электроснабжения в промышленности:
    • электродвигатели;
    • индукционные печи;
    • дроссели приборов освещения;
    • сварочные трансформаторы.

    Это явление в обязательном порядке необходимо учитывать при дальнейших расчетах. В более мощных электроприборах нагрузка идет гораздо больше, поэтому в расчетах коэффициент мощности принимают 0,8.

    При подсчете нагрузки на бытовые приборы запас мощности нужно брать 5%. Для электросети этот процент становит 20%.

    Оцените качество подачи материала: (3

    Источник: https://DomStrouSam.ru/raschet-moshhnosti-po-toku-i-napryazheniyu-shema-i-tablitsyi/

    Калькулятор электрической мощности

    Этот калькулятор электрической мощности помогает вычислить мощность, потребляемую электрическими устройствами. Хотите узнать, как рассчитать электрическую мощность? Вам интересно: какой коэффициент мощности? Просто прочтите текст ниже, чтобы узнать.

    Как рассчитать электрическую мощность

    Электрическая мощность, как и механическая мощность, — это количество работы, выполняемой за единицу времени. В электрических схемах работа выполняется электрическим током.Мощность зависит от «количества рабочих, доступных в единицу времени» — тока I и энергии «одного рабочего» — напряжения В . В цепи постоянного тока мощность

    P = I * V ,

    где

    • I [A] — ток,
    • В [В] — напряжение,
    • P [Вт] — мощность.

    В цепях переменного тока уравнение

    P = I * V * PF ,

    , где новый символ PF означает коэффициент мощности .

    Какой коэффициент мощности

    В переменном токе и ток, и напряжение периодически меняются во времени. Значения I или V соответствуют среднеквадратическому значению (RMS). RMS — это квадратный корень из среднего квадратов чисел. Обычно упоминаемое напряжение электрических розеток ( 230 В в ЕС и Австралии, 110 В в США и Канаде, 100 В в Японии) является среднеквадратичным напряжением. В цепях переменного тока ток и напряжение могут не совпадать по фазе.Максимальное значение тока может опережать или отставать от максимального значения напряжения. Это делает передачу мощности менее эффективной. В худшем случае, когда ток и напряжение полностью не синхронизированы, передаваемая мощность равна нулю.

    Коэффициент мощности показывает, насколько синхронизирован ток с напряжением. Если они синхронизированы, коэффициент мощности составляет 1 . В противном случае оно меньше единицы, достигая нуля в случае полной рассинхронизации. Коэффициент мощности зависит от устройства.Для чисто резистивного устройства, такого как электрический чайник или электронагреватель, коэффициент мощности составляет 1 . Устройство с индуктивными или емкостными элементами выводит ток и напряжение из фазы. Это делает его коэффициент мощности меньше 1. Проверьте калькулятор коэффициента мощности, чтобы узнать больше.

    Калькулятор электрической мощности

    Для вычисления электрической мощности необходимо указать ток, напряжение и коэффициент мощности. Для устройств, подключенных к розеткам, напряжение равно напряжению бытовой электросети.Ток, потребляемый устройством, обычно можно найти либо на вилке, либо где-нибудь на устройстве. Коэффициент мощности найти немного сложнее — если у вас под рукой нет анализатора качества электроэнергии. В этом списке указаны коэффициенты мощности нескольких типичных бытовых устройств:

    • лампы со стандартной колбой: PF = 1 ,
    • люминесцентные лампы: PF = 0,93 ,
    • Обычный асинхронный двигатель
    • при половинной нагрузке: PF = 0,73 , при полной нагрузке: PF = 0.85 ,
    • электрическая духовка (с резистивным нагревательным элементом): PF = 1.0 ,
    • индукционная печь: PF = 0,85 .

    Точное значение коэффициента мощности зависит от деталей конструкции, поэтому относитесь к этим значениям с недоверием.

    Электроэнергетика и энергия | Физика

    Цели обучения

    К концу этого раздела вы сможете:

    • Рассчитайте мощность, рассеиваемую резистором, и мощность, подаваемую источником питания.
    • Рассчитайте стоимость электроэнергии при различных обстоятельствах.

    Мощность в электрических цепях

    Мощность ассоциируется у многих с электричеством. Зная, что мощность — это коэффициент использования или преобразования энергии, каково выражение для электроэнергии ? На ум могут прийти линии электропередач. Мы также думаем о лампочках с точки зрения их номинальной мощности в ваттах. Сравним лампочку на 25 Вт с лампой на 60 Вт.(См. Рис. 1 (а).) Поскольку оба работают от одного и того же напряжения, лампа мощностью 60 Вт должна потреблять больше тока, чтобы иметь большую номинальную мощность. Таким образом, сопротивление лампы на 60 Вт должно быть ниже, чем у лампы на 25 Вт. Если мы увеличиваем напряжение, мы также увеличиваем мощность. Например, когда лампочка мощностью 25 Вт, рассчитанная на работу от 120 В, подключена к 240 В, она на короткое время очень ярко светится, а затем перегорает. Как именно напряжение, ток и сопротивление связаны с электроэнергией?

    Рис. 1. (a) Какая из этих лампочек, лампа мощностью 25 Вт (вверху слева) или лампа мощностью 60 Вт (вверху справа), имеет более высокое сопротивление? Что потребляет больше тока? Что потребляет больше всего энергии? Можно ли по цвету сказать, что нить накаливания мощностью 25 Вт круче? Является ли более яркая лампочка другого цвета, и если да, то почему? (кредиты: Дикбаух, Wikimedia Commons; Грег Вестфолл, Flickr) (б) Этот компактный люминесцентный светильник (КЛЛ) излучает такую ​​же интенсивность света, как и лампа мощностью 60 Вт, но при входной мощности от 1/4 до 1/10.(кредит: dbgg1979, Flickr)

    Электрическая энергия зависит как от напряжения, так и от перемещаемого заряда. Проще всего это выражается как PE = qV , где q — это перемещенный заряд, а V — это напряжение (или, точнее, разность потенциалов, через которую проходит заряд). Мощность — это скорость перемещения энергии, поэтому электрическая мощность равна

    .

    [латекс] P = \ frac {PE} {t} = \ frac {qV} {t} \\ [/ latex].

    Учитывая, что ток равен I = q / t (обратите внимание, что Δ t = t здесь), выражение для мощности принимает вид

    P = IV

    Электрическая мощность ( P ) — это просто произведение тока на напряжение.Мощность имеет знакомые единицы ватт. Поскольку единицей СИ для потенциальной энергии (PE) является джоуль, мощность выражается в джоулях в секунду или ваттах. Таким образом, 1 A ⋅V = 1 Вт. Например, в автомобилях часто есть одна или несколько дополнительных розеток, с помощью которых можно заряжать сотовый телефон или другие электронные устройства. {2} R \\ [/ latex].

    Обратите внимание, что первое уравнение всегда верно, тогда как два других можно использовать только для резисторов. В простой схеме с одним источником напряжения и одним резистором мощность, подаваемая источником напряжения, и мощность, рассеиваемая резистором, идентичны. (В более сложных схемах P может быть мощностью, рассеиваемой одним устройством, а не полной мощностью в цепи.) Из трех различных выражений для электрической мощности можно получить различное понимание. Например, P = V 2 / R подразумевает, что чем ниже сопротивление, подключенное к данному источнику напряжения, тем больше подаваемая мощность.Кроме того, поскольку напряжение возведено в квадрат в P = V 2 / R , эффект от приложения более высокого напряжения, возможно, больше, чем ожидалось. Таким образом, когда напряжение увеличивается вдвое до лампочки мощностью 25 Вт, ее мощность увеличивается почти в четыре раза до примерно 100 Вт, что приводит к ее перегоранию. Если бы сопротивление лампы оставалось постоянным, ее мощность была бы ровно 100 Вт, но при более высокой температуре ее сопротивление также будет выше.

    Пример 1. Расчет рассеиваемой мощности и тока: горячая и холодная энергия

    (a) Рассмотрим примеры, приведенные в Законе Ома: сопротивление и простые цепи и сопротивление и удельное сопротивление.Затем найдите мощность, рассеиваемую автомобильной фарой в этих примерах, как в горячую, так и в холодную погоду. б) Какой ток он потребляет в холодном состоянии?

    Стратегия для (а)

    Для горячей фары нам известны напряжение и ток, поэтому мы можем использовать P = IV , чтобы найти мощность. Для холодной фары нам известны напряжение и сопротивление, поэтому мы можем использовать P = V 2 / R , чтобы найти мощность.

    Решение для (a)

    Вводя известные значения тока и напряжения для горячей фары, получаем

    P = IV = (2.{2}} {0,350 \ text {} \ Omega} = 411 \ text {W} \\ [/ latex].

    Обсуждение для (а)

    30 Вт, рассеиваемые горячей фарой, являются типичными. Но 411 Вт в холодную погоду на удивление выше. Начальная мощность быстро уменьшается по мере увеличения температуры лампы и увеличения ее сопротивления.

    Стратегия и решение для (b)

    Ток при холодной лампочке можно найти несколькими способами. Переставляем одно из уравнений мощности, P = I 2 R , и вводим известные значения, получая

    [латекс] I = \ sqrt {\ frac {P} {R}} = \ sqrt {\ frac {411 \ text {W}} {{0.350} \ text {} \ Omega}} = 34,3 \ text {A} \\ [/ latex].

    Обсуждение для (б)

    Холодный ток значительно выше, чем установившееся значение 2,50 А, но ток будет быстро снижаться до этого значения по мере увеличения температуры лампы. Большинство предохранителей и автоматических выключателей (используемых для ограничения тока в цепи) спроектированы так, чтобы выдерживать очень высокие токи на короткое время при включении устройства. В некоторых случаях, например, с электродвигателями, ток остается высоким в течение нескольких секунд, что требует использования специальных плавких предохранителей с замедленным срабатыванием.

    Чем больше электроприборов вы используете и чем дольше они остаются включенными, тем выше ваш счет за электроэнергию. Этот знакомый факт основан на соотношении энергии и мощности. Вы платите за использованную энергию. Поскольку P = E / t , мы видим, что

    E = Pt

    — это энергия, используемая устройством, использующим мощность P в течение временного интервала t . Например, чем больше горит лампочек, тем больше используется P ; чем дольше они включены, тем больше т .Единицей измерения энергии в счетах за электричество является киловатт-час (кВт ч), что соответствует соотношению E = Pt . Стоимость эксплуатации электроприборов легко оценить, если у вас есть некоторое представление об их потребляемой мощности в ваттах или киловаттах, времени их работы в часах и стоимости киловатт-часа для вашей электросети. Киловатт-часы, как и все другие специализированные единицы энергии, такие как пищевые калории, можно преобразовать в джоули. Вы можете доказать себе, что 1 кВт ⋅ ч = 3.6 × 10 6 Дж.

    Потребляемая электрическая энергия ( E ) может быть уменьшена либо за счет сокращения времени использования, либо за счет снижения энергопотребления этого прибора или приспособления. Это не только снизит стоимость, но и снизит воздействие на окружающую среду. Улучшение освещения — один из самых быстрых способов снизить потребление электроэнергии в доме или на работе. Около 20% энергии в доме расходуется на освещение, в то время как в коммерческих учреждениях эта цифра приближается к 40%.Флуоресцентные лампы примерно в четыре раза эффективнее ламп накаливания — это верно как для длинных ламп, так и для компактных люминесцентных ламп (КЛЛ). (См. Рис. 1 (b).) Таким образом, лампу накаливания мощностью 60 Вт можно заменить на КЛЛ мощностью 15 Вт, которая имеет такую ​​же яркость и цвет. КЛЛ имеют изогнутую трубку внутри шара или спиралевидную трубку, соединенную со стандартным резьбовым основанием, подходящим для стандартных розеток лампы накаливания. (В последние годы были решены исходные проблемы с цветом, мерцанием, формой и высокими начальными вложениями в КЛЛ.) Теплопередача от этих КЛЛ меньше, и они служат до 10 раз дольше. В следующем примере рассматривается важность инвестиций в такие лампы. Новые белые светодиодные лампы (которые представляют собой группы небольших светодиодных лампочек) еще более эффективны (в два раза больше, чем у КЛЛ) и служат в 5 раз дольше, чем КЛЛ. Однако их стоимость по-прежнему высока.

    Установление соединений: энергия, мощность и время

    Отношение E = Pt может оказаться полезным во многих различных контекстах.Энергия, которую ваше тело использует во время упражнений, зависит, например, от уровня мощности и продолжительности вашей активности. Количество нагрева от источника питания зависит от уровня мощности и времени его применения. Даже доза облучения рентгеновского изображения зависит от мощности и времени воздействия.

    Пример 2. Расчет рентабельности компактных люминесцентных ламп (КЛЛ)

    Если стоимость электроэнергии в вашем районе составляет 12 центов за кВтч, какова общая стоимость (капитальные плюс эксплуатация) использования лампы накаливания мощностью 60 Вт в течение 1000 часов (срок службы этой лампы), если стоимость лампы составляет 25 центов? (б) Если мы заменим эту лампочку компактной люминесцентной лампой, которая дает такой же световой поток, но составляет четверть мощности и стоит 1 доллар.50, но длится в 10 раз дольше (10 000 часов), какова будет общая стоимость?

    Стратегия

    Чтобы найти эксплуатационные расходы, мы сначала находим используемую энергию в киловатт-часах, а затем умножаем ее на стоимость киловатт-часа.

    Решение для (a)

    Энергия, используемая в киловатт-часах, определяется путем ввода мощности и времени в выражение для энергии:

    E = Pt = (60 Вт) (1000 ч) = 60,000 Вт ⋅ ч

    В киловатт-часах это

    E = 60.0 кВт ⋅ ч.

    Сейчас стоимость электроэнергии

    Стоимость

    = (60,0 кВт ч) (0,12 долл. США / кВт час) = 7,20 долл. США.

    Общая стоимость составит 7,20 доллара за 1000 часов (около полугода при 5 часах в день).

    Решение для (b)

    Поскольку CFL использует только 15 Вт, а не 60 Вт, стоимость электроэнергии составит 7,20 доллара США / 4 = 1,80 доллара США. КЛЛ прослужит в 10 раз дольше, чем лампа накаливания, так что инвестиционные затраты составят 1/10 стоимости лампы за этот период использования, или 0.1 (1,50 доллара США) = 0,15 доллара США. Таким образом, общая стоимость 1000 часов составит 1,95 доллара США.

    Обсуждение

    Следовательно, использование КЛЛ намного дешевле, даже несмотря на то, что первоначальные вложения выше. Повышенная стоимость рабочей силы, которую бизнес должен включать в себя для более частой замены ламп накаливания, здесь не учитывается.

    Подключение: Эксперимент на вынос — Инвентаризация использования электроэнергии

    1) Составьте список номинальной мощности для ряда приборов в вашем доме или комнате.Объясните, почему что-то вроде тостера имеет более высокий рейтинг, чем цифровые часы. Оцените энергию, потребляемую этими приборами в среднем за день (оценивая время их использования). Некоторые приборы могут указывать только рабочий ток. Если бытовое напряжение 120 В, то используйте P = IV . 2) Проверьте общую мощность, используемую в туалетах на этаже или в здании вашей школы. (Возможно, вам придется предположить, что используемые длинные люминесцентные лампы рассчитаны на 32 Вт.) Предположим, что здание было закрыто все выходные, и что эти огни были оставлены включенными с 6 часов вечера.{2} R \\ [/ латекс].

    • Энергия, используемая устройством с мощностью P за время t , составляет E = Pt .

    Концептуальные вопросы

    1. Почему лампы накаливания тускнеют в конце своей жизни, особенно незадолго до того, как их нити оборвутся?

    Мощность, рассеиваемая на резисторе, равна P = V 2 / R , что означает, что мощность уменьшается при увеличении сопротивления. Однако эта мощность также определяется соотношением P = I 2 R , что означает, что мощность увеличивается при увеличении сопротивления.Объясните, почему здесь нет противоречия.

    Задачи и упражнения

    1. Какова мощность разряда молнии 1,00 × 10 2 МВ при токе 2,00 × 10 4 A ?

    2. Какая мощность подается на стартер большого грузовика, который потребляет 250 А тока от аккумуляторной батареи 24,0 В?

    3. Заряд в 4,00 Кл проходит через солнечные элементы карманного калькулятора за 4,00 часа. Какова выходная мощность, если выходное напряжение вычислителя равно 3.00 В? (См. Рисунок 2.)

    Рис. 2. Полоса солнечных элементов прямо над клавишами этого калькулятора преобразует свет в электричество для удовлетворения своих потребностей в энергии. (Источник: Эван-Амос, Wikimedia Commons)

    4. Сколько ватт проходит через него фонарик с 6,00 × 10 2 за 0,500 ч использования, если его напряжение составляет 3,00 В?

    5. Найдите мощность, рассеиваемую каждым из этих удлинителей: (a) удлинительный шнур с сопротивлением 0,0600 Ом, через который 5.00 А течет; (б) более дешевый шнур с более тонким проводом и сопротивлением 0,300 Ом.

    6. Убедитесь, что единицами измерения вольт-ампер являются ватты, как следует из уравнения P = IV .

    7. Покажите, что единицы 1V 2 / Ω = 1W, как следует из уравнения P = V 2 / R .

    8. Покажите, что единицы 1 A 2 Ω = 1 Вт, как следует из уравнения P = I 2 R .

    9. Проверьте эквивалент единиц энергии: 1 кВт ч = 3,60 × 10 6 Дж.

    10. Электроны в рентгеновской трубке ускоряются до 1,00 × 10 2 кВ и направляются к цели для получения рентгеновских лучей. Вычислите мощность электронного луча в этой трубке, если она имеет ток 15,0 мА.

    11. Электрический водонагреватель потребляет 5,00 кВт за 2,00 часа в сутки. Какова стоимость его эксплуатации в течение одного года, если электроэнергия стоит 12,0 центов / кВт · ч? См. Рисунок 3.

    Рисунок 3. Водонагреватель электрический по запросу. Тепло в воду подается только при необходимости. (кредит: aviddavid, Flickr)

    12. Сколько электроэнергии необходимо для тостера с тостером мощностью 1200 Вт (время приготовления = 1 минута)? Сколько это стоит при 9,0 цента / кВт · ч?

    13. Какова будет максимальная стоимость КЛЛ, если общая стоимость (капиталовложения плюс эксплуатация) будет одинаковой как для КЛЛ, так и для ламп накаливания мощностью 60 Вт? Предположим, что стоимость лампы накаливания составляет 25 центов, а электричество стоит 10 центов / кВтч.Рассчитайте стоимость 1000 часов, как в примере с КЛЛ по рентабельности.

    14. Некоторые модели старых автомобилей имеют электрическую систему 6,00 В. а) Каково сопротивление горячему свету у фары мощностью 30,0 Вт в такой машине? б) Какой ток протекает через него?

    15. Щелочные батареи имеют то преимущество, что они выдают постоянное напряжение почти до конца своего срока службы. Как долго щелочная батарея с номиналом 1,00 А · ч и 1,58 В будет поддерживать горение лампы фонарика мощностью 1,00 Вт?

    16.Прижигатель, используемый для остановки кровотечения в хирургии, выдает 2,00 мА при 15,0 кВ. а) Какова его выходная мощность? б) Какое сопротивление пути?

    17. В среднем телевизор работает 6 часов в день. Оцените ежегодные затраты на электроэнергию для работы 100 миллионов телевизоров, предполагая, что их потребляемая мощность составляет в среднем 150 Вт, а стоимость электроэнергии составляет в среднем 12,0 центов / кВт · ч.

    18. Старая лампочка потребляет всего 50,0 Вт, а не 60,0 Вт из-за истончения ее нити за счет испарения.Во сколько раз уменьшается его диаметр при условии равномерного утонения по длине? Не обращайте внимания на любые эффекты, вызванные перепадами температур.

    Медная проволока калибра 19. 00 имеет диаметр 9,266 мм. Вычислите потери мощности в километре такого провода, когда он пропускает 1,00 × 10 2 A.

    Холодные испарители пропускают ток через воду, испаряя ее при небольшом повышении температуры. Одно такое домашнее устройство рассчитано на 3,50 А и использует 120 В переменного тока с эффективностью 95,0%.а) Какова скорость испарения в граммах в минуту? (b) Сколько воды нужно налить в испаритель за 8 часов работы в ночное время? (См. Рисунок 4.)

    Рис. 4. Этот холодный испаритель пропускает ток непосредственно через воду, испаряя ее напрямую с относительно небольшим повышением температуры.

    21. Integrated Concepts (a) Какая энергия рассеивается разрядом молнии с током 20 000 А, напряжением 1,00 × 10 2 МВ и длиной 1.00 мс? (б) Какую массу древесного сока можно было бы поднять с 18ºC до точки кипения, а затем испарить за счет этой энергии, если предположить, что сок имеет те же тепловые характеристики, что и вода?

    22. Integrated Concepts Какой ток должен вырабатывать подогреватель бутылочек с питанием от батарей 12,0 В, чтобы нагреть 75,0 г стекла, 250 г детской смеси и 3,00 × 10 2 алюминия от 20 ° C до 90º за 5,00 мин?

    23. Integrated Concepts Сколько времени требуется хирургическому прижигателю для повышения температуры 1.00 г ткани от 37º до 100, а затем закипятите 0,500 г воды, если она выдает 2,00 мА при 15,0 кВ? Не обращайте внимания на передачу тепла в окружающую среду.

    24. Integrated Concepts Гидроэлектрические генераторы (см. Рисунок 5) на плотине Гувера вырабатывают максимальный ток 8,00 × 10 3 A при 250 кВ. а) Какая выходная мощность? (b) Вода, питающая генераторы, входит и покидает систему с низкой скоростью (таким образом, ее кинетическая энергия не изменяется), но теряет 160 м в высоте.Сколько кубических метров в секунду необходимо при КПД 85,0%?

    Рисунок 5. Гидроэлектрические генераторы на плотине Гувера. (кредит: Джон Салливан)

    25. Integrated Concepts (a) Исходя из 95,0% эффективности преобразования электроэнергии двигателем, какой ток должны обеспечивать аккумуляторные батареи на 12,0 В 750-килограммового электромобиля: отдых до 25,0 м / с за 1,00 мин? (b) Подняться на холм высотой 2,00 × 10 2 м за 2,00 мин при постоянной 25.Скорость 0 м / с при приложении силы 5,00 × 10 2 Н для преодоления сопротивления воздуха и трения? (c) Двигаться с постоянной скоростью 25,0 м / с, прилагая силу 5,00 × 10 2 Н для преодоления сопротивления воздуха и трения? См. Рисунок 6.

    Рис. 6. Электромобиль REVAi заряжается на одной из улиц Лондона. (кредит: Фрэнк Хебберт)

    26. Integrated Concepts Пригородный легкорельсовый поезд потребляет 630 А постоянного тока напряжением 650 В при ускорении.а) Какова его мощность в киловаттах? (b) Сколько времени нужно, чтобы достичь скорости 20,0 м / с, начиная с состояния покоя, если его масса в загруженном состоянии составляет 5,30 × 10 4 кг, при условии эффективности 95,0% и постоянной мощности? (c) Найдите его среднее ускорение. (г) Обсудите, как ускорение, которое вы обнаружили для легкорельсового поезда, сравнивается с тем, что может быть типичным для автомобиля.

    27. Integrated Concepts (a) Линия электропередачи из алюминия имеет сопротивление 0,0580 Ом / км. Какова его масса на километр? б) Какова масса на километр медной линии с таким же сопротивлением? Более низкое сопротивление сократит время нагрева.Обсудите практические ограничения ускорения нагрева за счет снижения сопротивления.

    28. Integrated Concepts (a) Погружной нагреватель, использующий 120 В, может повысить температуру 1,00 × 10 2 -г алюминиевой чашки, содержащей 350 г воды, с 20 ° C до 95 ° C за 2,00 мин. Найдите его сопротивление, предполагая, что оно постоянно в процессе. (b) Более низкое сопротивление сократит время нагрева. Обсудите практические ограничения ускорения нагрева за счет снижения сопротивления.

    29. Integrated Concepts (a) Какова стоимость нагрева гидромассажной ванны, содержащей 1500 кг воды, от 10 ° C до 40 ° C, исходя из эффективности 75,0% с учетом передачи тепла в окружающую среду? Стоимость электроэнергии 9 центов / кВт kWч. (b) Какой ток потреблял электрический нагреватель переменного тока 220 В, если на это потребовалось 4 часа?

    30 . Необоснованные результаты (a) Какой ток необходим для передачи 1,00 × 10 2 МВт мощности при 480 В? (b) Какая мощность рассеивается линиями передачи, если они имеют коэффициент 1.00 — сопротивление Ом? (c) Что неразумного в этом результате? (d) Какие допущения необоснованны или какие посылки несовместимы?

    31. Необоснованные результаты (a) Какой ток необходим для передачи мощности 1,00 × 10 2 МВт при 10,0 кВ? (b) Найдите сопротивление 1,00 км провода, которое вызовет потерю мощности 0,0100%. (c) Каков диаметр медного провода длиной 1,00 км, имеющего такое сопротивление? (г) Что необоснованного в этих результатах? (e) Какие предположения необоснованны или какие посылки несовместимы?

    32.Создайте свою проблему Рассмотрим электрический погружной нагреватель, используемый для нагрева чашки воды для приготовления чая. Постройте задачу, в которой вы рассчитываете необходимое сопротивление нагревателя, чтобы он увеличивал температуру воды и чашки за разумный промежуток времени. Также рассчитайте стоимость электроэнергии, используемой в вашем технологическом процессе. Среди факторов, которые необходимо учитывать, — это используемое напряжение, задействованные массы и теплоемкость, тепловые потери и время, в течение которого происходит нагрев.Ваш инструктор может пожелать, чтобы вы рассмотрели тепловой предохранительный выключатель (возможно, биметаллический), который остановит процесс до того, как в погружном блоке будут достигнуты опасные температуры.

    Глоссарий

    электрическая мощность:
    скорость, с которой электрическая энергия подается источником или рассеивается устройством; это произведение тока на напряжение

    Избранные решения проблем и упражнения

    1. 2,00 × 10 12 Вт

    5.{6} \ text {J} \\ [/ latex]

    11. 438 $ / год

    13. $ 6.25

    15. 1.58 ч

    17. 3,94 миллиарда долларов в год

    19. 25,5 Вт

    21. (а) 2,00 × 10 9 Дж (б) 769 кг

    23. 45.0 с

    25. (а) 343 A (б) 2,17 × 10 3 A (в) 1,10 × 10 3 A

    27. (а) 1,23 × 10 3 кг (б) 2,64 × 10 3 кг

    29. (a) 2,08 × 10 5 A
    (b) 4,33 × 10 4 МВт
    (c) Линии передачи рассеивают больше мощности, чем они должны передавать.
    (d) Напряжение 480 В неоправданно низкое для напряжения передачи. В линиях передачи на большие расстояния поддерживается гораздо более высокое напряжение (часто сотни киловольт), чтобы уменьшить потери мощности.

    Расчет энергии и мощности в электрических цепях — Видео и стенограмма урока

    Energy and Power

    Когда вы включаете эти электрические цепи, вы можете увидеть их энергию и мощность в действии. Когда вы щелкаете выключателем света в своей комнате, вы видите, как ваш свет становится ярким и начинает излучать свет.Когда вы включаете телевизор, вы видите, что экран на вашем телевизоре включается, и вы можете видеть различные телевизионные программы, предоставленные вам местными телевизионными провайдерами. Когда вы включаете компьютер, вы видите, что ваш монитор включается, а затем вы можете использовать на нем все виды компьютерных программ. Когда вы включаете свой мобильный телефон, вы можете звонить и писать текстовые сообщения своим друзьям и семье

    Откуда эта сила и энергия? Он поступает либо от батарей, либо от вашего поставщика электроэнергии.Поскольку мы имеем дело с электрическими цепями, мы также имеем дело с электроэнергией. Мы определяем электрическая мощность как скорость, с которой схема использует электрическую энергию. Электрическая энергия — это заряд батареи или электростанции. Есть два способа рассчитать эту мощность. Мы можем использовать либо напряжение, либо сопротивление цепи.

    Использование напряжения

    Чтобы использовать напряжение цепи, мы можем использовать эту формулу для расчета мощности:

    P = В * I

    У нас есть эта мощность равна напряжению цепь умножает на ток I цепи.Когда наша единица измерения напряжения — вольты ( В, ), а единица измерения тока — амперы или, для краткости, амперы ( A ), тогда мы должны умножить их вместе, чтобы получить ватт ( Вт, ), стандартная единица власти. Предположим, у нас есть две 2,5-вольтовые батареи, соединенные вместе, чтобы получить 5 вольт. Если мы пропустим через него ток 0,5 ампера, мы получим мощность:

    P = 5 В * 0,5 A = 2,5 Вт

    Вы можете запитать часы с 2.5 Вт мощности.

    Использование сопротивления

    Другой способ рассчитать мощность — использовать величину сопротивления в цепи. Вы можете фактически преобразовать формулу мощности с напряжением в формулу мощности с сопротивлением, используя закон Ома, который говорит вам, что В = I * R (напряжение равно току, умноженному на сопротивление).

    P = V * I

    P = ( I * R ) * I

    P =
    0 I ^ 2 * R ^ 2 * R ,

    P = V * I

    P = V * ( V / R )

    P =
    018 V
    ^ 2/ Если вам заданы ток и сопротивление, вы можете использовать P = I ^ 2 * R , чтобы найти свою мощность.2/192 Ом = 14400/192 = 75 Вт

    Этого достаточно для питания швейной машины.

    Итоги урока

    Давайте рассмотрим, что мы узнали. Электрическая цепь представляет собой замкнутый контур, по которому свободно течет электричество. Электрическая мощность — это скорость, с которой схема использует электрическую энергию. Формула для определения мощности в цепи:

    P = V * I

    Мы используем закон Ома ( V = I * R ), чтобы получить две другие формулы мощности, которые используйте сопротивление цепи.2/ R

    Электроэнергия — Веб-формулы

    Электрическая мощность определяется по формуле:
    P = V · I
    Где V — напряжение, а I — ток.

    Соответствующие единицы:
    ватт (Вт) = вольт (В) · ампер (A)


    Мощность также можно определить по следующим формулам:
    P = I 2 · R R = P / I 2 I
    906 R )
    P = V 2 / R R = V 6 9016 9016 9016 9016 V = √ ( P · R )


    Подробнее об Electric Power 9061 1
    Электроэнергия определяется как скорость, с которой работа выполняется источником эл.м.ф. в поддержании тока в электрической цепи. Практическая единица мощности — киловатт и лошадиные силы; где 1 киловатт = 100 ватт и 1 л.с. = 746 ватт.

    Если сопротивления (например, электрические приборы) соединены последовательно, ток через каждое сопротивление будет одинаковым. Тогда мощность электрического прибора, P α R и P α V (поскольку V = IR), это означает, что в комбинации сопротивлений серии разность потенциалов и потребляемая мощность будут больше при большем сопротивлении .

    Если сопротивления ( i.е. электроприборов) подключены параллельно, разность потенциалов на каждом приборе одинакова. Тогда P α 1 / R и I α 1 / R (как V = IR), что означает, что в параллельной комбинации сопротивлений потребляемый ток и мощность будут больше при меньшем сопротивлении.

    Для данного напряжения В, , если сопротивление изменилось с R на ( R / n ), а потребляемая мощность изменилась с P на nP , затем согласно P = V 2 / R , имеем:


    P = V 2 / (R / n)) = n (V 2 / R) = nP, где R = R / n и P = nP

    Когда приборы питания P 1 , P 2 , P 3 P n включены последовательно с источником напряжения, эффективная потребляемая мощность ( P с ) определяется по формуле:


    1/ P с = 1 / P 1 + 1 / P 2 + 1 / P 3 +… + 1 / P n
    Для приборов n , каждый из сопротивление R , соединены последовательно с источником напряжения В, рассеиваемая мощность P s тогда определяется как:
    (1) P s = V 2 / n R

    Когда приборы питания

    P 1 , P 2 , P 3 P n подключены параллельно к источнику напряжения, эффективная мощность потреблено ( P p ) затем определяется следующим образом:
    P s = P 1 + P 2 + P 3 +… + P n Для приборов n , каждое с равным сопротивлением R , подключено параллельно к источнику напряжения В , рассеиваемая мощность тогда определяется как:
    (2) P p = В 2 / ( R / n) = n V 2 / R

    Из (1) и (2) мы имеем P p / P s = n 2 или просто записывается как : P p = n 2 P s .

    В соответствии с приведенными выше формулами мы можем объяснить, что:


    При группировке ламп
    серии
    по заданному источнику напряжения лампа большей мощности будет давать меньшую яркость и будет иметь меньший потенциал сопротивления на ней, но тот же ток , тогда как в параллельном группировке лампочек через данный источник напряжения, лампа большей мощности даст большую яркость и позволит большему току проходить через нее, но будет иметь меньшее сопротивление и такую ​​же разность потенциалов на нем.

    Электроэнергия
    Электроэнергия определяется как общая выполненная работа или энергия, поставленная источником ЭДС. при поддержании тока в электрической цепи в течение заданного времени:
    Электрическая энергия = электрическая мощность × время = P × t

    Таким образом, формула для электрической энергии имеет вид:
    Электрическая энергия = P × t = V × I × t = I 2 × R × t = V 2 t / R

    S.I единица электрической энергии — джоуль (обозначается Дж), где 1 джоуль = 1 ватт × 1 секунда = 1 вольт × 1 ампер × 1 секунда
    Коммерческая единица электрической энергии — киловатт-час ( кВт · ч, ), где 1 кВтч = 1000 Вт h = 3,6 × 10 6 Дж = одна единица потребляемой электроэнергии .

    Количество единиц потребляемой электроэнергии равно n = (общая мощность × время в часе) / 1000
    Стоимость потребления электроэнергии в доме = количество.единиц потребленной электроэнергии × количество на одну единицу электроэнергии.

    Теорема о максимальной мощности
    В ней говорится, что выходная мощность источника тока максимальна, когда внутреннее сопротивление источника равно внешнему сопротивлению в цепи. Итак, если R — внешнее сопротивление цепи, а r — внутреннее сопротивление источника тока (то есть батареи), то выходная мощность максимальна, когда R = R.

    Эта теорема применима ко всем типам источников ЭДС. и связан с выходной мощностью, а НЕ с рассеиваемой мощностью.

    Если E — применяемая ЭДС. источника ЭДС. т.е. . батарея с внутренним сопротивлением r и R — внешнее сопротивление, тогда ток в цепи определяется как:
    I = E / (R + r)

    При максимальной выходной мощности R = r , поэтому имеем:
    I = E / (r + r) = E / (2r)
    и
    максимальная выходная мощность:
    P max = I 2 r = E 2 / (4r)

    При коротком замыкании аккумулятора мощность равна нулю.В этом случае вся мощность батареи рассеивается внутри батареи из-за ее внутреннего сопротивления. Таким образом, мощность, рассеиваемая внутри батареи, определяется как: P = ( E / r) 2 × r = E 2 / r

    КПД источника ЭДС.
    КПД источника ЭДС. определяется как отношение выходной мощности (, т. е. , мощность на внешнем сопротивлении цепи, к входной мощности (т. е.мощность, потребляемая от источника ЭДС). Итак,

    Где V = падение потенциала на внешнем сопротивлении R,
    E = E.M.F. источника тока,
    I = ток в цепи.

    Если r — внутреннее сопротивление источника ЭДС, тогда
    В = IR и E = I (R + r )
    или

    Когда мощность, полученная от источника, максимальная, тогда R = р. В данной ситуации имеем:

    Таким образом максимальная эффективность источника эл.м.ф. составляет 50%. Это означает, что для элемента только половина общей мощности, потребляемой элементом, используется для полезных целей, тогда как другая половина рассеивается внутри элемента.

    Пример 1:
    Лифт должен поднимать 1000 кг на расстояние 100 м со скоростью 4 м / с. Какую в среднем мощность оказывает лифт во время этой поездки?
    Решение:
    Работу, проделанную лифтом на 100 метров, легко вычислить:
    W = mgh = (1000) (9.8) (100) = 9,8 × 10 5 Джоулей.

    Общее время поездки можно рассчитать по скорости лифта:
    t = x / v = 100 м / 4 м / с = 25 с .

    Таким образом, средняя мощность определяется по формуле: P = Вт / t = 9,8 × 10 5 / 25s = 3,9 × 10 4 Вт или 39 кВт.

    Пример 2:
    Считается, что объект в свободном падении достиг конечной скорости , если сопротивление воздуха становится достаточно сильным, чтобы противодействовать всему ускорению свободного падения, в результате чего объект падает с постоянной скоростью.Точное значение конечной скорости зависит от формы объекта, но для многих объектов оно может быть оценено на уровне 100 м / с. Когда объект весом 10 кг достиг предельной скорости, какую силу сопротивление воздуха оказывает на объект?

    Решение: Для решения этой проблемы мы будем использовать уравнение P = Fv cos θ , Вместо обычного уравнения мощности, поскольку нам дана скорость объекта. Нам просто нужно вычислить силу, прилагаемую к объекту сопротивлением воздуха, и угол между силой и скоростью объекта.Поскольку объект достиг постоянной скорости, результирующая сила, действующая на него, должна быть равна нулю. Поскольку на объект действуют только две силы: сила тяжести и сопротивление воздуха, сопротивление воздуха должно быть равным по величине и противоположным по направлению силе тяжести. Таким образом, F a = — F G = мг = 98 N, направленный вверх. Таким образом, сила, прикладываемая сопротивлением воздуха, антипараллельна скорости объекта. Таким образом:
    P = Fv cos θ = (98) (100) (cos180) = — 9800 Вт

    Пример 3: Мощность двигателя насоса составляет 4 кВт.Сколько воды в кг / мин он может поднять на высоту 20 м? (g = 10 м / с 2 )
    Решение:
    Заданная мощность двигателя P = 4KW = 4000 Вт
    Если масса воды, поднятая за одну секунду, = m кг.
    Общий объем работы, выполненной при подъеме воды, W = mgh
    Мощность P = Вт / т, но t = 1 минута = 60 сек.
    4000 = mgh / 60
    4000 = (m × 10 × 20) / 60
    m = 1200 кг.

    Пример 4 : Когда вода течет по трубе, ее скорость изменяется на 5%, найти изменение силы воды?
    Решение: Мощность = Сила × Скорость = Скорость изменения количества движения × скорость = {(масса / время) × скорость} x скорость = {(adv) × v} × v = adv 3 где «a» — площадь поперечного сечения, «d» — плотность воды, а «v» — скорость потока воды.
    Следовательно, Сила воды прямо пропорциональна кубу скорости воды, поэтому пусть
    P = Kv 3 (k — постоянная величина, равная ad.)
    Взятие бревна с обеих сторон
    log P = 3log v + log k
    Дифференциация с обеих сторон
    dP / P = 3dv / v
    процентное изменение мощности, dP / P × 100 = 3 × 5% = 15%.

    Пример 5 : Кинетическая энергия выбрасываемой воды из плотины используется для вращения турбины. Труба, по которой устремляется вода — 2.4 метра и его скорость 12 м / сек. Предполагая, что вся кинетическая энергия воды используется для вращения турбины, вычислите производимый ток, если эффективность динамо-машины составляет 60% и станция передает мощность 240 кВ. Плотность воды = 10 3 кг / м 3 .
    Решение: Учитывая, что
    r = радиус трубы = 1,2 м, средняя скорость воды v = 12 м / с
    V = 240 кВ = 240 × 10 3 вольт, плотность воды p = 10 3 кг / м 3 .
    Теперь кинетическая энергия текущей воды в секунду, т.е.
    Power P = (1/2) (массовый расход в секунду) × v 2
    = (1/2) pr 2 (l / t) rv 2
    = (1/2) pr 2 rv 3
    = (1/2) 3,14 × (1,2) 2 × 10 3 × (12) 3 Вт
    = 3,9 x 10 6 Вт

    Ток в кабелях передачи определяется по формуле:
    ток = выходная мощность / напряжение
    = (60% мощности P) / (240 × 1000)
    = [(60/100) × 3.9 × 10 6 ] / (240 × 1000) = 9,75 A

    Электроэнергия, работа и мощность

    Чтобы понять, как работают устойчивые технологии, важно усвоить определенные основные принципы. Знать, как фотоэлектрические элементы преобразуют солнечную энергию в электричество, означает понимать основы электричества и света. Понимание того, как ветряные турбины производят электричество, означает понимание кое-чего о мощности, работе и электромагнетизме.В этом модуле будут представлены основные концепции, необходимые для понимания технологий, обсуждаемых в этом курсе. Хотя формулы иногда используются для объяснения основных принципов, суть не в том, чтобы уметь решать количественные проблемы. Формулы помогут вам увидеть взаимосвязь.

    Цели обучения: Студенты смогут:

    1. Выделите различия между энергией, работой и мощностью и приведите примеры каждого из них с использованием соответствующих единиц.
    2. Дайте соответствующие определения для следующих электрических терминов: электрон, электрический заряд, электрический потенциал, сопротивление, ток, мощность, проводник, полупроводник и изолятор.

      Учащийся сможет сопоставить электрические величины / свойства с различными единицами измерения, используемыми в электротехнике (например, вольт, ампер, ватт, ом, ампер-часы, киловатт-часы и т. Д.).

    3. Обозначить элементы электрической цепи.
    4. Укажите различия между параллельными и последовательными цепями и отметьте влияние на электрический потенциал (измеренный в вольтах) и ток (измеренный в амперах).
    5. Объясните взаимосвязь между потоком тока и магнетизмом и покажите, как это лежит в основе электродвигателей и генераторов.
    6. Различайте электричество постоянного и переменного тока, определите полезные качества каждого из них, отметьте, какие устройства связаны с каждым из них, и опишите роль силовых инверторов.

    Энергия, работа и власть

    Перейти к: Force | Работа | Мощность

    Проще говоря, Вселенная состоит из четырех вещей: пространства, времени, массы и энергии.Первый закон термодинамики гласит, что энергия не может быть ни создана, ни разрушена. Но Эйнштейн показал нам, что энергию можно превратить в массу и наоборот. Второй закон термодинамики гласит, что каждый раз, когда энергия меняет форму, часть ее превращается в тепло. Энергия бывает разных форм. Самая полезная энергия или энергия высочайшего качества — это то, что мы можем использовать для работы. Например, энергия движения (кинетическая энергия) воды, падающей через плотину, может быть использована для вращения водяного колеса для измельчения зерна или выработки электричества.

    Потенциальная и кинетическая энергия

    Provenance: Первоисточник: Environment Canada (https://www.ec.gc.ca/eau-water/default.asp?lang=en&n=00EEE0E6-1), доступ через USGS: https://water.usgs .gov / edu / wuhy.html Это воспроизведение является копией официальной работы, опубликованной правительством Канады, и воспроизведение не было произведено в сотрудничестве или с одобрения правительства Канады.
    Повторное использование: Информация на этом веб-сайте была размещена с намерением сделать ее доступной для личного или общественного некоммерческого использования и может быть воспроизведена частично или полностью и любыми средствами без взимания платы или дополнительного разрешения, если не указано иное.Пользователи должны: проявлять должную осмотрительность для обеспечения точности воспроизводимых материалов; Укажите как полное название воспроизводимых материалов, так и организацию автора; и Укажите, что воспроизведение является копией официального произведения, опубликованного правительством Канады, и что воспроизведение не было произведено при поддержке или с одобрения правительства Канады.

    Самая низкая форма энергии с точки зрения полезности — тепло.Да, тепло можно использовать для производства пара и привода электрических турбин. Но для этого требуется много тепла, и это тепло должно исходить от какого-то другого источника энергии, например, горящего угля или солнечного света. Физики используют термин энтропия, чтобы описать изменение полезной энергии на менее полезное тепло.

    Проще говоря, вселенная состоит из четырех вещей; пространство, время, масса и энергия. Первый закон термодинамики гласит, что энергия не может быть ни создана, ни разрушена. (Хотя позже Эйнштейн показал, что для ядерных реакций энергию можно превратить в массу и наоборот).Энергия бывает разных форм. Когда энергия передается от одного объекта к другому или когда она преобразуется из одного типа в другой, ее можно использовать для выполнения работы. Например, энергия движения (кинетическая энергия) воды, падающей через плотину, может быть использована для вращения водяного колеса для измельчения зерна или выработки электричества.

    Энтропия — это мера распределения энергии. Концентрированные формы энергии, такие как энергия, хранящаяся в ядре атома, в химических связях или в высоковольтных электрических устройствах, очень полезны для выполнения работы.С другой стороны, менее концентрированные формы энергии, такие как низкотемпературное тепло, вибрации или звуковые волны, гораздо менее полезны. Второй закон термодинамики гласит, что всякий раз, когда энергия используется для выполнения работы, часть энергии превращается из концентрированной формы в менее полезную. Физики говорят, что по мере того, как энергия распространяется или рассеивается, энтропия увеличивается. Одним из результатов второго закона термодинамики является то, что ни один процесс не может преобразовать 100% энергии в полезную работу.

    Что такое энергия? Полезно разделить энергию на два списка. Кинетическая энергия — это энергия движущегося объекта. Падающая вода (реагирующая на силу тяжести), солнечный свет, электроны, протекающие по проводу (электричество), велосипед в движении, использование мускулов для движения глаз во время чтения — все это примеры кинетической энергии. Потенциальная энергия — это то, что сохраняется и готово к преобразованию в кинетическую энергию. Это включает воду, удерживаемую плотиной, электрический заряд, хранящийся в батарее, химическую энергию, хранящуюся в жирах и сахарах, и химическую энергию, хранящуюся в бензине и угле.

    На схеме гидроэлектростанции вода, стекающая по напорному штоку, имеет кинетическую энергию. Эта кинетическая энергия используется для вращения турбины, соединенной с электрическим генератором. Вода, хранящаяся за плотиной, имеет потенциальную энергию или запасенную энергию. Обратите внимание, что сила тяжести, действующая на воду, в каждом случае обеспечивает энергию.

    Сила

    Когда к объекту прикладывается энергия, мы думаем об этом как о силе .Некоторые силы требуют контакта между двумя объектами, а другие действуют на расстоянии. Силы, которые требует контакта , включают толкание, тянущее усилие (натяжение) и трение. Силы, которые действуют без прямого контакта между объектами, включают гравитацию, магнетизм и электрическую силу. Стандартная единица силы названа в честь сэра Исаака Ньютона, отца физики. Один Ньютон (1 Н) = количество силы для ускорения 1 кг массы на один метр в секунду 2 . Или 1 Н = (1 кг x 1 м) / с 2 .

    Аппарат Джоуля для демонстрации эквивалентности работы и тепла

    Provenance: Изображение из нового ежемесячного журнала Harper’s, № 231, август 1869 г. Доступно по: https://commons.wikimedia.org/wiki/File:Joule%27s_Apparatus_(Harper%27s_Scan).png
    Повторное использование: Этот элемент является общественным достоянием и может использоваться повторно без ограничений.

    Работа

    Мы используем энергию для работы. Самый простой способ думать о работе — это перемещать объект.Когда к объекту прикладывается сила (масса, умноженная на ускорение), которая заставляет этот объект перемещаться, пройденное расстояние — это работа, которая выполняется. Но мы используем энергию для выполнения большего количества работ, чем перемещение мебели или автомобилей. Работа также выполняется, когда мы используем солнечный свет или природный газ для обогрева наших домов, когда мы используем электричество для освещения наших комнат или когда мы используем бутерброд с арахисовым маслом и желе для питания клеток нашего мозга.

    Поскольку энергия бывает разных форм, неудивительно, что существуют разные способы ее измерения.Трудно отслеживать все различные единицы энергии. Посмотрите на таблицу ниже, чтобы увидеть некоторые единицы и отношение к джоулям, который является золотым стандартом измерения энергии. Он назван в честь Джеймса Джоуля, пивовара 19-го века, который показал эквивалентность механической работы и тепла. Один джоуль примерно равен количеству энергии, необходимому для поднятия 100-граммового яблока на 1 метр (3,3 фута).

    Изображенный аппарат был использован Джеймсом Джоулем для демонстрации эквивалентности механической работы и тепла.Он рассчитал работу, выполняемую силой тяжести на гирю. Эта тяга повернула лопаточные колеса, которые смешали воду в изолированном контейнере. Вода нагревается при перемешивании, показывая, что тепло = работа.

    Паровая машина Ватта

    Provenance: Wikicommons: https://commons.wikimedia.org/wiki/File:SteamEngine_Boulton%26Watt_1784.png
    Повторное использование: Этот элемент находится в общественном достоянии и может использоваться повторно без ограничений.

    Мощность

    Мощность — это мера того, сколько энергии используется за определенный период времени. Для этого мы можем использовать ватт. Джеймс Ватт был пионером в понимании физики энергии и разработал один из первых успешных паровых двигателей. Он одолжил нам свою фамилию для этого подразделения.

    Показано изображение паровой машины, разработанной совместно Джеймсом Ваттом для откачки воды из затопленных угольных шахт в Англии.

    Ватт — это один джоуль энергии, затрачиваемый за секунду. Таким образом, ватт включает в себя как затраченную энергию, так и время, в течение которого она была затрачена.По аналогии, вы можете получить один галлон воды из капающего крана за один час или из открытого крана за 15 секунд. В конце концов, вы все равно получите галлон воды, но во втором случае вода течет в ведро намного быстрее. Так что аспект времени важен. Мы используем термин мощность для обозначения количества энергии и скорости ее доставки. Джоуль — это член энергии, а ватт — член мощности.

    Насколько велик ватт мощности? Подбрасывание 100 г яблока в воздух на 1 м (3.3 фута) потребляет 1 ватт мощности. Ноутбук, который вы можете использовать для чтения, потребляет около 5 & acirc; & # 128; & # 147; 50 ватт, в зависимости от того, работает ли у вас в фоновом режиме музыка или работают другие приложения. Старомодная лампа накаливания мощностью 100 Вт потребляет 1 киловатт-час электроэнергии, если оставить ее включенной на 10 часов. Киловатт — это 1000 ватт, сокращенно кВт. 10 часов x 100 Вт = 1000 кВтч. Обратите внимание на разницу между кВт и кВтч. КВт — это мера мощности, а кВтч — мера того, сколько энергии было использовано в целом.

    Яблоко, падающее на метр, делает это с мощностью 1 ватт.

    Происхождение: Эван-Амос Автор изображения
    Повторное использование: Лицо, связавшее произведение с этим документом, посвятило произведение общественному достоянию, отказавшись от всех своих прав на произведение во всем мире в соответствии с законом об авторском праве, включая все смежные и смежные права в пределах, разрешенных законом. Вы можете копировать, изменять, распространять и выполнять работу даже в коммерческих целях, не спрашивая разрешения

    Вы не понимаете, что такое кВт и кВтч? Это уловка.Помните, что ватт — это джоуль / сек. Значит, в ватт или киловатт уже заложено время. Это энергия / время. Это мощность, скорость использования энергии. Но мощность не сообщает вам, сколько энергии было использовано за определенный период времени. Чтобы получить это, вам нужно умножить мощность на время. Затем единицы времени должны быть зачеркнуты. Увы, принято оставлять час на месте — глупо, но так оно и делается. 1 кВтч = 1 кВт x 1 час.

    Вот пример. В моем доме есть фотоэлектрическая система (солнечная электроэнергия), которая в идеальных условиях приятного солнечного прохладного дня рассчитана на выработку 4 кВт.За 4 часа это составит:

    4 кВт x 4 часа = 16 кВт · ч электроэнергии. В частично облачный день система может работать на половинной мощности или на 2 кВт выходной мощности. При такой скорости мне потребуется 8 часов, чтобы выработать те же 16 кВт · ч, что я сделал в солнечный день; 2кВт x 8 часов = 16 кВтч.

    В состоянии покоя типичный человек использует энергию 80 Вт для обеспечения жизненных функций организма (так называемый метаболизм в состоянии покоя). Взрослый мужчина может съедать около 2000 килокалорий в день. Одна ккал = 1,163 Вт · ч. Таким образом, диета на 2000 ккал обеспечит 2326 Втч или 2 Втч.326 кВтч. Если бы человек просто пролежал в постели 24 часа, он бы сжег 80 Вт x 24 часа = 1920 Вт · ч или 1 920 кВт · ч. Если этот парень останется в постели и продолжит так же есть, он в конечном итоге потребляет 2,326 кВтч & acirc; & # 128; & # 147; 1,920 кВтч = 0,406 кВтч больше, чем он использует, и это будет храниться в виде жира. Фунт жира равен примерно 3500 ккал (4 070,5 кВтч). Так что через десять дней он может прибавить еще фунт. Интенсивная поездка на велосипеде использует энергию в размере 200 Вт. Поэтому ему следует подумать о двухчасовой поездке на велосипеде, чтобы оставаться в форме (0.2 кВт для езды на велосипеде x 2 часа = 4,0 кВтч).

    Сводка силы, работы и мощности

    Сила = энергия, приложенная к объекту (измеряется в ньютонах).

    Работа = Сила X Расстояние или количество переданного тепла (Измеряется в Джоулях или калориях) .

    Мощность = работа / время (измеряется в ваттах, с)

    Различные единицы энергии

    1 калория (термохимическая) = 4.184 Дж

    1 британская тепловая единица = 251,9958 калорий

    1 БТЕ (термохимический) = 1054,35 Дж

    1 киловатт-час (кВтч) = 3,6 x 106 Дж

    1 киловатт-час (кВтч) = 3412 британских тепловых единиц (IT)

    1 терм = 100 000 британских тепловых единиц

    1 электрон-вольт = 1,6022 x 10-19 Дж

    Электричество и магнетизм

    Изолированные провода

    Происхождение: Чатама размещено на Викискладе https://commons.wikimedia.org/wiki/File:600V_CV_5.5sqmm.jpg
    Повторное использование: Этот файл находится под лицензией Creative Commons Attribution-Share Alike 3.0 Непортированная лицензия. Вы можете: делиться — копировать, распространять и передавать произведение для ремикса — адаптировать произведение При следующих условиях: приписывание — вы должны атрибутировать работу способом, указанным автором или лицензиаром (но ни в коем случае не предполагает, что они одобряют вас или ваше использование произведения). совместно использовать — если вы изменяете, трансформируете или расширяете эту работу, вы можете распространять полученную работу только по той же или аналогичной лицензии, что и эта.

    Теперь, когда у вас есть хорошее представление об энергии, работе и мощности, пора зарядиться и изучить электричество! Древние имели смутное представление об электричестве из-за своего жизненного опыта.Рыбаки, ловившие разного рода «электрическую рыбу», при обращении с ней подвергались шоку. Другие чувствовали воздействие статического электричества от своей шерстяной одежды. Египтяне видели связь между электрической рыбой и молнией. Но только около 1600 года начались серьезные научные исследования электричества. Усилиями многих исследователей к концу 19 века было разработано хорошее представление об электричестве и о том, как его использовать.

    Напомним, что вся материя состоит из атомов.А атомы состоят из нескольких основных частиц: электронов с отрицательным зарядом, протонов с положительным зарядом и нейтронов без заряда. Электричество можно представить как поток электронов через проводник, подобный медному проводу. На самом деле это не поток электронов, а импульс, который проходит по проводу.

    Хорошие проводники, как и металлы, легко пропускают электричество. У них есть электроны на внешних орбиталях, с которыми легко вступить в контакт. Плохие проводники называются изоляторами, и они не пропускают беспрепятственный ток электричества.Даже самые лучшие проводники оказывают некоторое сопротивление току электричества. Такое сопротивление измеряется в единицах, называемых Ом. Стекло — хороший изолятор и, следовательно, плохой проводник.

    Третий класс соединений — полупроводники. Они реагируют на изменение условий, чтобы включить или выключить подачу электричества. Полупроводники часто содержат смесь кремния и металлов. Пластины из этих полупроводников лежат в основе «микросхем» компьютера, а также являются основой для светодиодных ламп и фотоэлектрических (солнечных) элементов.

    Фотоэлектрические панели изготовлены из полупроводников.

    Происхождение: Фото Б. Цукера
    Повторное использование: бесплатно для повторного использования

    Панели фотоэлементов, которые производят электричество из солнечного света, сделаны из полупроводников.

    Для подачи электричества должна быть замкнутая цепь. Электроны должны начинать с состояния с высокой энергией и заканчиваться в состоянии с низкой энергией. Ниже представлена ​​схема простой схемы. Обратите внимание, что электричество проходит от высокоэнергетического конца батареи через лампу, а затем обратно к низкоэнергетическому концу батареи.Когда выключатель разомкнут, подача электричества прекращается.

    Об электричестве просто думать как об электроне (или импульсе размером с электрон), протекающем по проводнику. Но на практике один электрон слишком мал и несет слишком мало энергии, чтобы выполнять какую-либо реальную работу. Однако групповые потоки электронов могут вызвать сильный толчок! Кулон — это 6,24 × 10 18 электронов. А amp — это поток в один кулон в секунду через проводник. Таким образом, ампер измеряет скорость потока электричества.Мы называем поток электричества током.

    Не все электричество течет с одинаковой силой. Чтобы понять это, подумайте о давлении или силе воды, выходящей из трубы. Если труба прикреплена к резервуару наверху высокого здания, вода будет иметь гораздо большее давление, чем если бы резервуар был на 30 см выше трубы. То же самое и с электричеством. «Давление» электричества — это электрический потенциал. Электрический потенциал — это количество энергии, доступное для проталкивания каждой единицы заряда через электрическую цепь.Единицей измерения электрического потенциала является вольт. Вольт равен джоуля на кулон. Таким образом, если автомобильный аккумулятор имеет электрический потенциал 12 вольт, он может обеспечить 12 джоулей энергии на каждый кулон заряда, который он подает на стартер. Точно так же, если розетка в вашем доме имеет электрический потенциал 120 вольт, то она может обеспечить 120 джоулей энергии на каждый кулон заряда, который доставляется на устройство, подключенное к стене. (Примечание: величина «электрический потенциал» иногда называется несколькими разными именами, включая напряжение, разность потенциалов и электродвижущую силу.Для ясности мы всегда будем ссылаться на электрический потенциал, который измеряется в вольтах). Электроны высокого напряжения возвращаются в «основное состояние» с большей энергией, чем электроны низкого напряжения.

    А вольт — это сила, необходимая для перемещения одного ампер через проводник с сопротивлением 1 Ом .

    Вы думаете: «Кажется, существует связь между усилителями, вольтами и омами» & acirc; & # 128; & # 148; и ты прав! Электрический потенциал = ток x сопротивление.Это закон Ома, который обычно записывается как: E = I x R . E — электрический потенциал, измеренный в вольтах, I — ток, измеренный в амперах, а R — сопротивление, измеренное в омах.

    Электроны, проходящие через сопротивление проволоки, совершают работу. Действительно полезны два вида работы, выполняемой током. Если в проводе имеется большое сопротивление, большая часть работы будет выполняться в виде тепла. Подумайте об электрическом тостере, феном или обогревателе.

    Второй действительно важный вид работы, выполняемой током, протекающим через провод, — это создание магнитного поля.Надеюсь, в детстве вы играли с постоянными магнитами. Вы знаете, что у магнитов два полюса: один называется северным, а другой — южным. Это название связано с использованием магнитов в компасах для определения направления. Вы знаете, что одинаковые концы магнитов отталкиваются друг от друга, а противоположные концы притягиваются. Теперь, когда электрический ток течет по проводу, он становится похож на магнит в том смысле, что у него есть магнитное поле. Однако, в отличие от постоянных магнитов, магнитное поле можно отключить, остановив ток.Это свойство лежит в основе работы электродвигателей. Ток, проходящий через обмотки проводов в электродвигателе, вызывает включение магнетизма. Затем это заставляет двигатели вращаться, притягиваясь и толкаясь притяжением и отталкиванием электромагнитов.

    Работа, совершаемая током с течением времени, называется мощностью. Мощность измеряется в ваттах. Но вы это уже знаете! Напомним, что выше вы узнали, что обычный человек в состоянии покоя сжигает 80 Вт.

    На электричество;

    1 Ватт = 1 А x 1 Вольт.

    Уравнение можно переформулировать для расчета производимого тока;

    1 ампер = 1 ватт / 1 объем т.

    Подведем итоги.

    Ампер измеряет количество электричества, протекающего с течением времени (ток).

    Ом измерьте сопротивление потоку.

    Вольт измеряет количество энергии, доступной для проталкивания каждой единицы заряда.

    Ватт — это мера мощности или работы, которая выполняется с течением времени.

    Вы знаете, что закон Ома устанавливает связь между E, I и R. Но сколько работы уже сделано? Это выражается как Сила. Мощность = Электрический потенциал x Ток, или P = E x I. Эта формула указывает на то, что мощность зависит как от количества поставляемой электроэнергии, так и от силы, стоящей за ней. Например, небольшая солнечная панель может выдавать 18 вольт и 2 ампера. Его мощность составит 18 вольт x 2 ампера = 36 ватт. Теперь можно построить еще одну солнечную панель для выработки 9 вольт и 4 ампер.Его мощность составит 9 вольт x 4 ампера = 36 ватт. Так же, как и другой!

    Цепи

    Простая схема

    Происхождение: Бенджамин Кукер, Университет Хэмптона
    Повторное использование: Этот элемент предлагается по лицензии Creative Commons Attribution-NonCommercial-ShareAlike http://creativecommons.org/licenses/by-nc-sa/3.0/ Вы можете повторно использовать это для некоммерческих целей, если вы указываете авторство и предлагаете любые производные работы по аналогичной лицензии.

    Еще раз о простой схеме

    Происхождение: Бенджамин Кукер, Университет Хэмптона
    Повторное использование: Этот элемент предлагается по лицензии Creative Commons Attribution-NonCommercial-ShareAlike http://creativecommons.org/licenses/by-nc-sa/3.0/ Вы можете повторно использовать это для некоммерческих целей, если вы указываете авторство и предлагаете любые производные работы по аналогичной лицензии.

    Оборудование, производящее и использующее электричество, подключено в электрическую цепь.Оборудование может быть установлено как последовательно, так и параллельно. Посмотрите на схемы ниже, чтобы увидеть последствия использования последовательной и параллельной схем. Для фотоэлектрических (PV) элементов каждая ячейка может производить только около 0,6 вольт. Поскольку для большинства приложений требуется более высокое напряжение, фотоэлементы должны быть подключены последовательно для получения желаемых результатов.

    Последовательная схема

    Происхождение: Бенджамин Кукер, Университет Хэмптона
    Повторное использование: Этот элемент предлагается по лицензии Creative Commons Attribution-NonCommercial-ShareAlike http: // creativecommons.org / licenses / by-nc-sa / 3.0 / Вы можете повторно использовать этот элемент в некоммерческих целях при условии указания авторства и предложения любых производных работ по аналогичной лицензии.

    Параллельная схема

    Происхождение: Бенджамин Кукер, Университет Хэмптона
    Повторное использование: Этот элемент предлагается по лицензии Creative Commons Attribution-NonCommercial-ShareAlike http://creativecommons.org/licenses/by-nc-sa/3.0/ Вы можете повторно использовать это для некоммерческих целей, если вы указываете авторство и предлагаете любые производные работы по аналогичной лицензии.

    Электродвигатели и генераторы

    Магнитное поле вокруг провода, по которому течет ток

    Происхождение: Бенджамин Кукер, Университет Хэмптона
    Повторное использование: Этот элемент предлагается по лицензии Creative Commons Attribution-NonCommercial-ShareAlike http://creativecommons.org/licenses/by-nc-sa/3.0/ Вы можете повторно использовать это для некоммерческих целей, если вы указываете авторство и предлагаете любые производные работы по аналогичной лицензии.

    Напомним, что часть работы, совершаемой электричеством, происходит, когда оно проходит через провод для создания магнитного поля.Ганс Кристиан Эрстед обнаружил это в 1820 году. Годом позже Майкл Фарадей показал, что магнитное поле вокруг провода можно использовать для создания электромагнитов, которые могут быть хитроумно скомпонованы для создания электродвигателя.
    Электромагнит

    Происхождение: Оригинальное фото Джины Клиффорд: https://www.flickr.com/photos/cobalt_grrl/2256696466
    Повторное использование: Attribution-ShareAlike 2.0 Generic (CC BY-SA 2.0) Бесплатно: Совместное использование — копирование и распространение материал на любом носителе или в любом формате. Адаптировать — ремикшировать, преобразовывать и дополнять материал для любых целей, даже в коммерческих целях.

    Обратите внимание на изображение электромагнита, полученное путем наматывания изолированного провода на железный гвоздь. Железный гвоздь концентрирует магнитное поле, создаваемое током в изолированном проводе. Изоляция предотвращает короткое замыкание цепи железным гвоздем.

    На схемах ниже показано, как работает электродвигатель. Обратите внимание, что при каждом половинном обороте контакты в коммутаторе меняют направление тока, чтобы двигатель вращался в том же направлении.

    Простой электродвигатель

    Происхождение: Изображения созданы или предоставлены для изучения.com защищены авторским правом © Chris Woodford (Объясните, что stuff.com) и опубликованы под этой лицензией Creative Commons. http://www.explainthatstuff.com/electricmotors.html
    Повторное использование: Per Creative Commons License: Совместное использование — копирование и распространение материала на любом носителе или любом формате. Адаптация — ремикс, преобразование и создание материала

    . Простой электродвигатель

    Происхождение: Создано Авинашем Синха в виде оригинального DIY-файла по лицензии Creative Commons на следующем веб-сайте: http: // www.Instructables.com/file/FW079IPGGC2UDG3/
    Повторное использование: По лицензии CC разрешено следующее: Совместное использование — копирование и распространение материала на любом носителе или любом формате. Адаптация — ремикс, преобразование и построение материала

    .
    Генератор постоянного тока

    Происхождение: Изображение с сайта www.alternative-energy-tutorials.com, используется с разрешения
    Повторное использование: Все учебные пособия и материалы, опубликованные и представленные на веб-сайте учебных пособий по альтернативным источникам энергии, включая текст, графику и изображения, являются собственностью авторских прав или аналогичных права Учебников по альтернативной энергии, представляющих www.Alternative-energy-tutorials.com, если прямо не указано иное. Согласно веб-мастеру AET: Как вы любезно спросили, я не возражаю против того, чтобы вы использовали это изображение как часть своего веб-курса по энергетике бесплатно. Тем не менее, я должен попросить вас правильно ссылаться на мои учебные пособия, изображения и сайт: www.alternative-energy-tutorials.com соответственно в своих презентациях.

    Майкл Фарадей не усовершенствовал электродвигатель, но он обнаружил важное свойство электромагнетизма, которое привело к другому великому изобретению — электрическому генератору.Фарадей открыл в 1831 году принцип магнитной индукции. Он обнаружил, что, проводя магнит по проводу, он вызывает электрический ток в замкнутой цепи. Это привело к разработке электрических генераторов. Первые успешные коммерческие разработки появились примерно в 1860 году. Электрогенератор — это, по сути, электродвигатель, который вращается под действием некоторой внешней силы и в ответ производит индуцированный ток. Гибридные электромобили, такие как Toyota Prius, делают именно это. Электродвигатель питается от аккумулятора при нажатии педали акселератора.Когда педаль отпускается, инерция автомобиля действует через вращающиеся колеса, чтобы вращать двигатель, заставляя двигатель работать в качестве генератора, создавая электричество для подзарядки аккумулятора.

    Электричество переменного и постоянного тока

    Генератор переменного тока

    Происхождение: Автор: Федеральное управление гражданской авиации http://www.faa.gov/regulations_policies/handbooks_manuals/aircraft/amt_handbook/media/FAA-8083-30_Ch20.pdf
    Повторное использование: Это изображение или файл являются работой Сотрудник Федерального управления гражданской авиации, взятый на работу или взятый на работу в рамках служебных обязанностей этого лица.Это произведение федерального правительства США, изображение находится в общественном достоянии Соединенных Штатов.

    До сих пор мы рассматривали только один вид электричества — постоянный ток (DC). Это то, что производят батареи, солнечные панели и генераторы постоянного тока. Для электричества постоянного тока ток всегда течет в одном и том же направлении. Другой вид электричества — это переменный ток (AC). Как видно из названия, ток переключает направление в проводе с регулярным циклом. Электроэнергия переменного тока — это то, что приходит в наши дома через электросеть.Производится генераторами переменного тока. Генератор переменного тока устроен иначе, чем генератор постоянного тока. Помните, что в генераторе постоянного тока или двигателе есть коммутатор или выпрямитель, который переключает направление тока в катушках якоря (той части, которая вращается). В генераторе переменного тока вместо реверсивного коммутатора используются контактные кольца. Таким образом, с каждой половиной оборота генератора индуцированный ток меняет направление.

    Выходной сигнал генератора переменного тока генерирует синусоидальную волну при скачках напряжения в цепи.Реверсирование тока происходит быстро. В Соединенных Штатах стандарт для электросети составляет 60 Гц (переключение вперед и назад 60 раз в секунду).

    Синусоидальная волна от генератора переменного тока

    Provenance: Booyabazooka в английской Википедии
    Повторное использование: Этот элемент предлагается по лицензии Creative Commons Attribution-NonCommercial-ShareAlike http://creativecommons.org/licenses/by-nc-sa/3.0/ Вы можете повторно использовать этот элемент в некоммерческих целях при условии указания авторства и предложения любых производных работ по аналогичной лицензии.

    На диаграмме справа показана синусоида, генерируемая генератором переменного тока. При напряжении выше 0 вольт электричество течет в одном направлении, а при напряжении ниже 0 вольт — в другом. Ось Y — напряжение, а ось X — время.

    Короткое видео о разнице между генераторами и двигателями постоянного и переменного тока

    Преимущество использования переменного тока заключается в том, что можно легко повышать или понижать напряжение в различных частях сети системы доставки. Это делают трансформаторы. Трансформатор состоит из двух расположенных бок о бок катушек, большой и малой.Обе катушки имеют общий железный сердечник. Переменный ток, проходящий через небольшую первичную катушку, за счет магнитной индукции создает ток более высокого напряжения в большей вторичной катушке. И обратное также верно: если первичная обмотка больше, вторичная обмотка меньшего размера будет иметь более низкое выходное напряжение.

    Трансформатор, используемый для увеличения переменного напряжения

    Происхождение: BillC в англоязычной Википедии
    Повторное использование: Выпущено под лицензией GNU Free Documentation License.

    Зачем вообще увеличивать и уменьшать напряжение? Помните, что V = I x R. Передача электричества на большие расстояния приводит к потере энергии на тепло из-за сопротивления проводов. Чтобы предотвратить это, напряжение увеличивается, что требует меньшего тока и меньших тепловых потерь. Когда вы подойдете к вашему дому, напряжение снова упадет. По высоковольтным линиям электропередачи может подаваться электроэнергия 765 кВ (то есть 765 000 вольт!). То, что получается от розетки, составляет 120 вольт.

    Переключение между переменным и постоянным током

    Инвертор для переключения с постоянного на переменный ток

    Происхождение: Фотография сделана Б.Cuker
    Повторное использование: Без копирования, можно использовать для любых целей.

    Поскольку мы используем электричество как переменного, так и постоянного тока, важно уметь преобразовывать одно в другое. Эту работу выполняет устройство, называемое инвертором мощности. Многие бытовые приборы работают от сети переменного тока. Холодильники, кондиционеры, лампы накаливания и люминесцентные лампы, пылесосы, фены и стиральные машины — все напрямую используют кондиционер. Электроника, такая как компьютеры, телевизоры и сотовые телефоны, требует постоянного тока.В устройствах обычно инвертор встроен в шнур питания переменного тока. По проводу, идущему от инвертора, проходит постоянный ток, необходимый устройству. Инверторы

    также могут использоваться для преобразования постоянного тока в переменный. Такие устройства позволяют использовать 12 В постоянного тока автомобиля для питания портативного компьютера. Дома, которые используют фотоэлектрические панели для использования солнечной энергии для производства электроэнергии, также должны преобразовывать свою выработку в соответствии с переменным током, если системы подключены к электросети.

    Оба типа инверторов используют электронные схемы для перехода на электричество.Теория их действия выходит за рамки этого основного устройства. Но вы должны знать, что силовые инверторы подчиняются второму закону термодинамики. Таким образом, в процессе преобразования энергия теряется на тепло. Но современные инверторы могут достигать КПД 95%.

    Показан силовой инвертор, который преобразует постоянный ток солнечных панелей в переменный ток для фотоэлектрической системы, подключенной к сети.

    Хранение и производство электроэнергии с помощью батарей

    Схема свинцово-кислотной батареи

    Provenance: Ohiostandard в английской Википедии — перенесено с en.wikipedia в Commons от Burpelson AFB с использованием CommonsHelper.
    Повторное использование: Разрешено копировать, распространять и / или изменять этот документ в соответствии с условиями лицензии GNU Free Documentation License версии 1.2 или любой более поздней версии, опубликованной Free Software Foundation; без неизменяемых разделов, без текстов на лицевой обложке и без текстов на задней обложке. Копия лицензии включена в раздел под названием GNU Free Documentation License.

    Батареи преобразуют потенциальную энергию химических веществ в кинетическую энергию электричества.Бенджамин Франклин ввел термин «батарея» для описания стопки стеклянных пластин с металлическим покрытием, которые он использовал для хранения энергии. Но то, что у него было, сегодня мы назвали бы конденсаторами. Батареи работают за счет соединения двух химических материалов, которые имеют разное сродство к электронам. Материалы анода предпочитают терять электроны, а материалы катода — получать их. Электроды батареи погружены в раствор, содержащий положительно и отрицательно заряженные ионы, называемый электролитом. При включении в цепь электроны текут от анода к катоду.В то же время отрицательно заряженные ионы в электролите перемещаются от катода к аноду для поддержания нейтральности заряда и, таким образом, замыкают электрическую цепь.

    В перезаряжаемой батарее реакции на аноде и катоде можно обратить вспять, используя электрическую энергию для подачи тока, который толкает электроны в противоположном направлении — от катода к аноду. Это восстанавливает исходное состояние двух электродов. Ваш портативный компьютер, мобильный телефон и автомобильный аккумулятор — все это примеры аккумуляторных батарей.В современных батареях используются комбинации различных типов металлов и соединений оксидов металлов, образованные из таких элементов, как углерод, кадмий, кобальт, литий, марганец, никель, свинец и цинк для повышения производительности.

    Батарея из лимона

    Происхождение: Тереза ​​Нотт из Викимедиа: https://commons.wikimedia.org/wiki/File:Lemon_battery.png
    Повторное использование: Этот файл находится под лицензией Creative Commons Attribution-Share Alike 3.0 Unported.Вы можете: делиться — копировать, распространять и передавать произведение для ремикса — адаптировать произведение При следующих условиях: приписывание — вы должны атрибутировать работу способом, указанным автором или лицензиаром (но ни в коем случае не предполагает, что они одобряют вас или ваше использование произведения). совместно использовать — если вы изменяете, трансформируете или расширяете эту работу, вы можете распространять полученную работу только по той же или аналогичной лицензии, что и эта.

    Простая батарея, использующая кислотный фрукт и два разных металла (бронза и стальные сплавы).

    Exercises Exercises for Module 1 (Microsoft Word 2007 (.docx) 17kB Jul12 17)

    1. Создайте цепь, используя две последовательно соединенные батареи и лампочку. Используйте цифровой мультиметр (DMM) для измерения электрического потенциала в вольтах между положительной и отрицательной клеммами в цепи. Теперь добавьте в цепь вторую лампочку последовательно с первой. Какова яркость каждой лампочки по сравнению с яркостью, когда в цепи была только одна лампочка? С помощью вольтметра измерьте напряжение между положительной клеммой аккумулятора и проводом сразу после первой лампочки, а затем сразу после второй лампочки.Запишите результаты. Теперь создайте цепь с двумя параллельными лампочками. Запишите яркость и напряжение на каждой лампочке.

    Объясните свои результаты.

    Простая схема с одной лампочкой

    Цепь с двумя последовательно включенными лампочками

    Цепь с двумя параллельно включенными лампочками

    2.Сделайте пять магнитов для выборщиков, каждый с проволокой разной длины, намотанной вокруг железных гвоздей: 10 см, 20 см, 30 см, 40 см и 50 см. В каждом случае на каждом конце провода должно быть по 10 см, чтобы его можно было подключить к батарее. Таким образом, катушка «10 см» будет фактически сделана из проволоки длиной 30 см и так далее. Подключите каждый магнит к батарее и прикрепите как можно больше канцелярских скрепок к магнитной цепочке с кончика ногтя. Запишите максимальное количество скрепок в каждом случае. Затем нарисуйте график зависимости максимального количества удерживаемых скрепок от длины провода, из которого сделаны обмотки.Объясните, почему график выглядит именно так.

    3. Соберите простой двигатель из предоставленного комплекта. Обязательно обратите внимание на инструкции о том, как удалить изоляцию на противоположных сторонах провода, который контактирует с зажимами аккумулятора.Когда вы заставите свой мотор вращаться, проведите следующие эксперименты.

    а. Обратите внимание на направление вращения двигателя. Можете ли вы заставить его пойти в обратном направлении? Объяснять.

    г. Теперь снимите магнит и переверните. Затем перезапустите мотор. Поворачивает ли он в том же направлении, что и раньше? Почему?

    г. Теперь переверните аккумулятор и перезапустите двигатель. Направление вращения осталось прежним? Объяснить, почему.

    г. Подумайте об электродвигателе как о системе.Определите источник энергии и судьбу этой энергии во вращающейся двигательной системе. В своем ответе используйте следующие термины: электрохимическая энергия, кинетическая энергия (энергия движения) и тепло. Нарисуйте созданную вами схему для запуска электродвигателя. Наденьте шляпу системного мышления.

    • Определите каждый компонент системы.
    • Отследите поток энергии через систему. Обязательно покажите, где он переходит от электрического тока к магнитной энергии, кинетической энергии и теплу.
    • Сделайте снимок вашей диаграммы и включите его в свой отчет.

    Является ли электродвигатель закрытой системой (вся энергия остается в системе) или это открытая система (некоторый обмен энергией с окружающей средой)?

    4. Из кусочка цитрусовых сделайте батарейку. Положите медный пенни с одной стороны фрукта и стальную скрепку с другой стороны. Измерьте напряжение с помощью цифрового мультиметра. Запишите результат: ______.

    Теперь попробуйте использовать фруктовый аккумулятор, чтобы зажечь светодиодную лампочку.Это работает? Объясните, что создает электричество.

    Список литературы

    Электромагниты и закон Фарадея

    Электродвигатель и генератор

    Асинхронный двигатель переменного тока

    Трансформаторы

    Преобразователи переменного / постоянного тока

    Как работают батареи

    Яркость лампы

    Падение напряжения (В)

    Первая лампочка

    Вторая лампа

    Яркость лампы

    Падение напряжения (В)

    Первая лампочка

    Вторая лампа

    Яркость лампы

    Падение напряжения (В)

    Длина провода в бухте (см)

    10

    20

    30

    40

    50

    Макс. нет. скрепок

    9.6: Электроэнергия и мощность

    ЦЕЛИ ОБУЧЕНИЯ

    К концу этого раздела вы сможете:

    • Выразить электрическую мощность через напряжение и ток
    • Опишите мощность, рассеиваемую резистором в электрической цепи
    • Расчет энергоэффективности и рентабельности приборов и оборудования

    В электрической цепи электрическая энергия непрерывно преобразуется в другие формы энергии.Например, когда в проводнике течет ток, электрическая энергия преобразуется в тепловую энергию внутри проводника. Электрическое поле, создаваемое источником напряжения, ускоряет свободные электроны, увеличивая их кинетическую энергию на короткое время. Эта увеличенная кинетическая энергия преобразуется в тепловую энергию в результате столкновений с ионами решетчатой ​​структуры проводника. Ранее мы определяли мощность как скорость выполнения работы силой, измеряемой в ваттах. Мощность также можно определить как скорость передачи энергии.В этом разделе мы обсуждаем скорость передачи энергии или мощности в электрической цепи.

    Мощность в электрических цепях

    Мощность ассоциируется у многих с электричеством. На ум могут прийти линии электропередач. Мы также думаем о лампочках с точки зрения их номинальной мощности в ваттах. Каково выражение для электроэнергии ?

    Давайте сравним лампу мощностью 25 Вт с лампой мощностью 60 Вт (рисунок \ (\ PageIndex {1a} \)). Лампа на 60 Вт светится ярче, чем лампа на 25 Вт.Хотя это не показано, лампа мощностью 60 Вт также теплее, чем лампа мощностью 25 Вт. Тепло и свет производятся путем преобразования электрической энергии. Кинетическая энергия, теряемая электронами при столкновениях, преобразуется во внутреннюю энергию проводника и излучения. Как напряжение, ток и сопротивление связаны с электроэнергией?

    Рисунок \ (\ PageIndex {1} \): (a) На изображении выше показаны две лампы накаливания: лампа мощностью 25 Вт (слева) и лампа мощностью 60 Вт (справа). Лампа мощностью 60 Вт обеспечивает более интенсивный свет, чем лампа мощностью 25 Вт.Электрическая энергия, подаваемая в лампочки, преобразуется в тепло и свет. (b) Эта компактная люминесцентная лампа (КЛЛ) излучает такой же свет, что и лампа мощностью 60 Вт, но при входной мощности от 1/4 до 1/10. (кредит a: модификация работ «Dickbauch» / Wikimedia Commons и Грега Вестфолла; кредит b: модификация работ «dbgg1979» / Flickr)

    Для расчета электрической мощности рассмотрите разницу напряжений, существующую на материале (рисунок \ (\ PageIndex {2} \)). Электрический потенциал \ (V_1 \) выше, чем электрический потенциал в \ (V_2 \), а разность напряжений отрицательна \ (V = V_2 — V_1 \).Как обсуждалось в разделе «Электрический потенциал», между двумя потенциалами существует электрическое поле, которое указывает от более высокого потенциала к более низкому. Напомним, что электрический потенциал определяется как потенциальная энергия на заряд, \ (V = \ Delta U / q \), и заряд \ (\ Delta Q \) теряет потенциальную энергию, перемещаясь через разность потенциалов.

    Рисунок \ (\ PageIndex {2} \): Когда есть разность потенциалов в проводнике, присутствует электрическое поле, которое указывает в направлении от более высокого потенциала к более низкому потенциалу.

    Если заряд положительный, на него действует сила электрического поля \ (\ vec {F} = m \ vec {a} = \ Delta Q \ vec {E} \). Эта сила необходима, чтобы заряд двигался. Эта сила не ускоряет заряд на всем расстоянии \ (\ Delta L \) из-за взаимодействия заряда с атомами и свободными электронами в материале. Скорость и, следовательно, кинетическая энергия заряда не увеличиваются в течение всего пути через \ (\ Delta L \), а заряд, проходящий через область \ (A_2 \), имеет ту же скорость дрейфа \ (v_d \), что и заряд, который проходит через область \ (A_1 \).Однако с зарядом работает электрическое поле, которое изменяет потенциальную энергию. Поскольку изменение разности электрических потенциалов отрицательное, электрическое поле оказывается равным

    .

    \ [E = — \ dfrac {(V_2 — V_1)} {\ Delta L} = \ dfrac {V} {\ Delta L}. \]

    Работа, совершаемая над зарядом, равна произведению электрической силы на длину приложения силы,

    \ [W = F \ Delta L = (\ Delta Q E) \ Delta L = \ left (\ Delta Q \ dfrac {V} {\ Delta L} \ right) \ Delta L = \ Delta Q V = \ Delta U.\]

    Заряд движется с дрейфовой скоростью \ (v_d \), поэтому работа, выполняемая над зарядом, приводит к потере потенциальной энергии, но средняя кинетическая энергия остается постоянной. Потерянная электрическая потенциальная энергия проявляется в материале как тепловая энергия. В микроскопическом масштабе передача энергии происходит из-за столкновений между зарядом и молекулами материала, что приводит к повышению температуры в материале. Потеря потенциальной энергии приводит к повышению температуры материала, который рассеивается в виде излучения.2 / R \), эффект от приложения более высокого напряжения, возможно, больше, чем ожидалось. Таким образом, когда напряжение увеличивается вдвое до лампочки мощностью 25 Вт, ее мощность увеличивается почти в четыре раза до примерно 100 Вт, что приводит к ее перегоранию. Если бы сопротивление лампы оставалось постоянным, ее мощность была бы ровно 100 Вт, но при более высокой температуре ее сопротивление также будет выше.

    Пример \ (\ PageIndex {1} \): Расчет мощности в электрических устройствах

    Двигатель лебедки постоянного тока рассчитан на 20,00 А при напряжении 115 В. Когда двигатель работает на максимальной мощности, он может поднимать объект весом 4900.00 N на расстояние 10,00 м, за 30,00 с, с постоянной скоростью.

    1. Какая мощность потребляет двигатель?
    2. С какой силой поднимается объект? Не обращайте внимания на сопротивление воздуха. (c) Предполагая, что разница в мощности, потребляемой двигателем, и мощности, используемой для подъема объекта, рассеивается в виде тепла за счет сопротивления двигателя, оценить сопротивление двигателя?

    Стратегия

    1. Мощность, потребляемая двигателем, может быть найдена с помощью \ (P = IV \).2 R \).

    Решение

    1. Мощность, потребляемая двигателем, равна \ (P = IV \), а ток равен 20,00 A, а напряжение составляет 115,00 В: \ [P = IV = (20,00 \, A) 115,00 \, V = 2300.00 \, W. \]
    2. Сила, используемая для подъема объекта, равна \ (P = Fv \), где сила равна весу объекта (1960 Н), а величина скорости равна \ [v = \ dfrac {10.00 \, m } {30,00 \, s} = 0,33 \ dfrac {m} {s} \] \ [P = Fv = (4900 \, N) 0.2R) \).

      Упражнение \ (\ PageIndex {1} \)

      Электродвигатели обладают достаточно высоким КПД. Двигатель мощностью 100 л.с. может иметь КПД 90%, а двигатель мощностью 1 л.с. может иметь КПД 80%. Почему важно использовать высокопроизводительные двигатели?

      Ответ

      Несмотря на то, что электродвигатели имеют высокий КПД, 10–20% потребляемой мощности тратится впустую, а не используется для выполнения полезной работы. Большая часть 10–20% потерянной мощности передается в тепло, рассеиваемое медными проводами, используемыми для изготовления катушек двигателя.Это тепло увеличивает тепло окружающей среды и увеличивает потребность электростанций, обеспечивающих электроэнергию. Спрос на электростанцию ​​может привести к увеличению выбросов парниковых газов, особенно если электростанция использует уголь или газ в качестве топлива.

      Предохранитель

      А (рисунок \ (\ PageIndex {3} \)) — это устройство, которое защищает цепь от слишком высоких токов. Предохранитель — это, по сути, короткий отрезок провода между двумя контактами. Как мы видели, когда ток проходит по проводнику, кинетическая энергия носителей заряда преобразуется в тепловую энергию в проводнике.Кусок проволоки в предохранителе находится под напряжением и имеет низкую температуру плавления. Проволока предназначена для нагрева и разрыва при номинальном токе. Предохранитель поврежден и подлежит замене, но он защищает остальную цепь. Предохранители срабатывают быстро, но есть небольшая задержка, пока провод нагревается и обрывается.

      Рисунок \ (\ PageIndex {3} \): Предохранитель состоит из отрезка провода между двумя контактами. Когда через провод проходит ток, превышающий номинальный, провод плавится, разрывая соединение.На фото — «перегоревший» предохранитель в месте обрыва провода, защищающего цепь (кредит: модификация работы «Шардайы» / Flickr).

      Автоматические выключатели также рассчитаны на максимальный ток и разомкнуты для защиты цепи, но могут быть сброшены. Автоматические выключатели реагируют намного быстрее. Работа автоматических выключателей выходит за рамки этой главы и будет обсуждаться в следующих главах. Еще один метод защиты оборудования и людей — прерыватель цепи замыкания на землю (GFCI), который широко используется в ванных комнатах и ​​кухнях.Торговые точки GFCI очень быстро реагируют на изменения тока. Эти выходы открываются при изменении магнитного поля, создаваемого токонесущими проводниками, что также выходит за рамки данной главы и рассматривается в следующей главе.

      Стоимость электроэнергии

      Чем больше электроприборов вы используете и чем дольше они остаются включенными, тем выше ваш счет за электроэнергию. Этот знакомый факт основан на соотношении энергии и мощности. Вы платите за использованную энергию.Поскольку \ (P = \ dfrac {dE} {dt} \), мы видим, что

      \ [E = \ int P dt \]

      — это энергия, используемая устройством, использующим мощность P в течение интервала времени t . Если мощность доставляется с постоянной скоростью, то энергия может быть найдена как \ (E = Pt \). Например, чем больше горело лампочек, тем больше использовалось P ; чем дольше они включены, тем больше т .

      Единицей измерения энергии в счетах за электричество является киловатт-час \ ((кВт \ cdot h) \), что соответствует соотношению \ (E = Pt \).6 \, J \).

      Потребляемую электрическую энергию ( E ) можно уменьшить либо за счет сокращения времени использования, либо за счет снижения энергопотребления этого прибора или приспособления. Это не только снижает стоимость, но и снижает воздействие на окружающую среду. Улучшение освещения — один из самых быстрых способов снизить потребление электроэнергии в доме или на работе. Около 20% энергии, потребляемой в доме, идет на освещение, а для коммерческих предприятий это число приближается к 40%.Флуоресцентные лампы примерно в четыре раза эффективнее ламп накаливания — это верно как для длинных ламп, так и для компактных люминесцентных ламп (КЛЛ), например, рисунок \ (\ PageIndex {1b} \). Таким образом, лампу накаливания мощностью 60 Вт можно заменить КЛЛ мощностью 15 Вт, которая имеет такую ​​же яркость и цвет. КЛЛ имеют изогнутую трубку внутри шара или спиралевидную трубку, соединенную со стандартным резьбовым основанием, подходящим для стандартных розеток лампы накаливания. (В последние годы были решены исходные проблемы с цветом, мерцанием, формой и высокими начальными вложениями в КЛЛ.)

      Теплопередача от этих КЛЛ меньше, и они служат до 10 раз дольше, чем лампы накаливания. В следующем примере рассматривается важность инвестиций в такие лампы. Новые белые светодиодные лампы (которые представляют собой группы небольших светодиодных лампочек) еще более эффективны (в два раза больше, чем у КЛЛ) и служат в пять раз дольше, чем КЛЛ.

      Пример \ (\ PageIndex {2 \): расчет рентабельности светодиодной лампы

      Типичная замена лампы накаливания мощностью 100 Вт — это светодиодная лампа мощностью 20 Вт.Светодиодная лампа мощностью 20 Вт может обеспечивать такое же количество света, как и лампа накаливания мощностью 100 Вт. Какова экономия затрат при использовании светодиодной лампы вместо лампы накаливания в течение одного года, если предположить, что 0,10 доллара за киловатт-час — это средний тариф на электроэнергию, взимаемый энергетической компанией? Предположим, что лампочка включена на три часа в день.

      Стратегия

      1. Рассчитайте энергию, используемую в течение года для каждой лампочки, используя \ (E = Pt \).
      2. Умножьте энергию на стоимость.

      Решение

      1. Рассчитайте мощность для каждой лампочки. \ [E_ {Incandescent} = Pt = 100 \, W \ left (\ dfrac {1 \, kW} {1000 \, W} \ right) \ left (\ dfrac {3 \, h} {day} \ right) (365 \, дни) = 109,5 \, кВт \ cdot ч \] \ [E_ {LED} = Pt = 20 \, W \ left (\ dfrac {1 \, kW} {1000 \, W} \ right) \ left (\ dfrac {3 \, h} {day} \ right) (365 \, days) = 21.9 \, кВт \ cdot h \]
      2. Рассчитайте стоимость для каждого. \ [cost_ {Incandescent} = 109,5 \, кВт \ cdot h \ left (\ dfrac {\ $ 0.10} {kW \ cdot h} \ right) = \ $ 10.95 \] \ [cost_ {LED} = 21.90 \, кВт \ cdot h \ left (\ dfrac {\ $ 0.10} {kW \ cdot h} \ right) = \ $ 2.19 \]

      Значение

      Светодиодная лампа потребляет на 80% меньше энергии, чем лампа накаливания, экономя 8,76 доллара по сравнению с лампой накаливания в течение одного года. Светодиодная лампа может стоить 20 долларов, а лампа накаливания мощностью 100 Вт может стоить 0,75 доллара, что следует учесть при расчетах. Типичный срок службы лампы накаливания составляет 1200 часов, а светодиодной лампы — 50 000 часов. Лампы накаливания хватило бы на 1.08 лет при 3 часах в день, а светодиодная лампа прослужит 45,66 года. Первоначальная стоимость светодиодной лампы высока, но стоимость для домовладельца составит 0,69 доллара за лампы накаливания по сравнению с 0,44 доллара за светодиодные лампы в год. (Обратите внимание, что светодиодные лампы дешевеют.) Экономия затрат в год составляет примерно 8,50 долларов США, и это только для одной лампы.

      Упражнение \ (\ PageIndex {2} \)

      Является ли эффективность различных лампочек единственным соображением при сравнении различных лампочек?

      Ответ

      Нет, эффективность — очень важный фактор для лампочек, но есть много других соображений.Как упоминалось выше, важными факторами являются стоимость лампочек и срок их службы. Например, лампы CFL содержат ртуть, нейротоксин, и их необходимо утилизировать как опасные отходы. При замене ламп накаливания, которые управляются диммером на светодиоды, может потребоваться замена диммера. Диммерные переключатели для светодиодных фонарей сопоставимы по цене с переключателями ламп накаливания, но это начальная стоимость, которую следует учитывать. Также следует учитывать спектр света, но существует широкий диапазон цветовых температур, поэтому вы сможете найти тот, который соответствует вашим потребностям.Ни одно из этих упомянутых соображений не предназначено для того, чтобы препятствовать использованию светодиодных или CFL лампочек, но они являются соображениями.

      Замена ламп накаливания на КЛЛ или светодиодные лампы — простой способ снизить потребление энергии в домах и на коммерческих объектах. Лампы CFL работают с совершенно другим механизмом, чем лампы накаливания. Механизм сложен и выходит за рамки данной главы, но здесь приводится очень общее описание механизма.Лампы CFL содержат пары аргона и ртути, заключенные в трубку спиральной формы. В лампах CFL используется «балласт», который увеличивает напряжение, используемое лампой CFL. Балласт производит электрический ток, который проходит через газовую смесь и возбуждает молекулы газа. Возбужденные молекулы газа излучают ультрафиолетовый (УФ) свет, который, в свою очередь, стимулирует флуоресцентное покрытие внутри трубки. Это покрытие флуоресцирует в видимом спектре, излучая видимый свет. Традиционные люминесцентные лампы и лампы CFL имели короткую временную задержку до нескольких секунд, пока смесь «нагревалась» и молекулы переходили в возбужденное состояние.Следует отметить, что эти лампы содержат ртуть, которая ядовита, но если лампа сломана, ртуть никогда не выделяется. Даже если колба сломана, ртуть имеет тенденцию оставаться во флуоресцентном покрытии. Количество также довольно невелико, и преимущество экономии энергии может перевесить недостаток использования ртути.

      Лампы CFL заменяются на светодиодные, где LED означает «светоизлучающий диод». Диод был кратко обсужден как неомический прибор, сделанный из полупроводникового материала, который позволяет току течь в одном направлении.Светодиоды — это особый тип диодов, изготовленных из полупроводниковых материалов, содержащих примеси в комбинациях и концентрациях, которые позволяют преобразовывать дополнительную энергию движения электронов во время электрического возбуждения в видимый свет. Полупроводниковые устройства будут объяснены более подробно в Физике конденсированного состояния.

      Коммерческие светодиоды быстро становятся стандартом для коммерческого и жилого освещения, заменяя лампы накаливания и КЛЛ. Они предназначены для работы в видимой области спектра и изготовлены из галлия, легированного атомами мышьяка и фосфора.Цвет, излучаемый светодиодом, зависит от материалов, используемых в полупроводнике, и от силы тока. В первые годы развития светодиодов маленькие светодиоды на печатных платах были красного, зеленого и желтого цветов, но теперь светодиодные лампочки можно запрограммировать на получение миллионов цветов света, а также множества различных оттенков белого света.

      Сравнение ламп накаливания, КЛЛ и светодиодных ламп

      Экономия энергии может быть значительной при замене лампы накаливания или лампы CFL на светодиодную.Лампочки оцениваются по количеству энергии, потребляемой лампочкой, а количество светового потока измеряется в люменах. Люмен (лм) — это производная от системы СИ единица светового потока и мера общего количества видимого света, излучаемого источником. Лампу накаливания мощностью 60 Вт можно заменить лампой CFL мощностью 13–15 Вт или светодиодной лампой мощностью 6–8 Вт, все три из которых имеют световой поток примерно 800 лм. Таблица светоотдачи для некоторых часто используемых лампочек представлена ​​в Таблице \ (\ PageIndex {1} \).

      Срок службы лампочек трех типов значительно различается. Срок службы светодиодной лампы составляет 50 000 часов, у CFL — 8 000 часов, а лампы накаливания — всего 1200 часов. Светодиодная лампа является самой прочной, легко выдерживает грубое обращение, такое как сотрясение и удары. Лампа накаливания плохо переносит такое же обращение, поскольку нить накаливания и стекло могут легко сломаться. Лампа CFL также менее долговечна, чем светодиодная лампа, из-за своей стеклянной конструкции.Количество выделяемого тепла составляет 3,4 БТЕ / ч для светодиодной лампы мощностью 8 Вт, 85 БТЕ / ч для лампы накаливания мощностью 60 Вт и 30 БТЕ / ч для лампы КЛЛ. Как упоминалось ранее, основным недостатком лампы CFL является то, что она содержит ртуть, нейротоксин, и ее необходимо утилизировать как опасные отходы. Из этих данных легко понять, почему светодиодные лампы быстро становятся стандартом в освещении.

      Таблица \ (\ PageIndex {1} \): Световой поток светодиодных ламп, ламп накаливания и CFL
      Световой поток (люмен) Светодиодные лампы (Вт) Лампы накаливания (Вт) Лампы CFL (Вт)
      450 4−5 40 9−13
      800 6-8 60 13−15
      1100 9−13 75 18−25
      1600 16−20 100 23-30
      2600 25−28 150 30-55
      Сводка отношений

      В этой главе мы обсудили взаимосвязь между напряжением, током, сопротивлением и мощностью.2R \). Хотя все возможные комбинации могут показаться ошеломляющими, не забывайте, что все они представляют собой комбинации всего двух уравнений: закона Ома \ ((V = IR) \) и степени \ ((P = IV) \).

      Авторы и авторство

      • Сэмюэл Дж. Линг (Государственный университет Трумэна), Джефф Санни (Университет Лойола Мэримаунт) и Билл Мобс со многими авторами. Эта работа лицензирована OpenStax University Physics в соответствии с лицензией Creative Commons Attribution License (4.0).

      Как рассчитать падение напряжения и потерю мощности в проводах

      Вы должны рассматривать провод как еще один последовательно включенный резистор.Вместо этого сопротивление \ $ \ text {R} _ {\ text {load}} \ $ подключено к источнику питания с напряжением \ $ \ text {V} \ $ …

      Вы должны увидеть это так: сопротивление \ $ \ text {R} _ {\ text {load}} \ $, подключенное к через два провода с сопротивлением \ $ \ text {R} _ {\ text {wire}} \ $ на блок питания с напряжением \ $ \ text {V} \ $:

      Теперь мы можем использовать \ $ \ text {V} = \ text {I} \ cdot {} \ text {R} \ $, где \ $ \ text {V} \ $ означает напряжение, \ $ \ text {I} \ $ для тока и \ $ \ text {R} \ $ для сопротивления.

      Пример

      Предположим, что напряжение, приложенное к цепи, равно \ $ 5 \ text {V} \ $. \ $ \ text {R} _ {\ text {load}} \ $ равно \ $ 250 \ Omega \ $, а сопротивление \ $ \ text {R} _ {\ text {wire}} \ $ равно \ $ 2.5 \ Omega \ $ (если вы не знаете сопротивление провода, см. ниже в разделе «Расчет сопротивления провода»). Сначала мы вычисляем ток в цепи, используя \ $ \ text {I} = \ dfrac {\ text {V}} {\ text {R}} \ $: \ $ \ text {I} = \ dfrac {5 } {250 + 2 \ cdot2.5} = \ dfrac {5} {255} = 0,01961 \ text {A} = 19.61 \ text {mA} \

      $

      Теперь мы хотим узнать, какое падение напряжения на одном куске провода используется \ $ \ text {V} = \ text {I} \ cdot {} \ text {R} \ $: \ $ \ text {V} = 0,01961 \ cdot2.5 = 0,049025 В = 49,025 \ text {мВ} \

      долл. США

      Таким же образом мы можем рассчитать напряжение в \ $ \ text {R} _ {\ text {load}} \ $: \ $ \ text {V} = 0.01961 \ cdot250 = 4.9025 \ text {V} \ $

      Предвидение потери напряжения

      Что, если нам действительно нужно напряжение \ $ 5 \ text {V} \ $ over \ $ \ text {R} _ {\ text {load}} \ $? Нам нужно будет изменить напряжение \ $ \ text {V} \ $ от источника питания, чтобы напряжение выше \ $ \ text {R} _ {\ text {load}} \ $ стало \ $ 5 \ text {V } \ $.

      Сначала мы вычисляем ток через \ $ \ text {R} _ {\ text {load}} \ $: \ $ \ text {I} _ {\ text {load}} = \ dfrac {\ text {V} _ {\ text {load}}} {\ text {R} _ {\ text {load}}} = \ dfrac {5} {250} = 0,02 \ text {A} = 20 \ text {mA} \

      долларов США

      Поскольку мы говорим о последовательном сопротивлении, ток во всей цепи одинаков. Следовательно, ток, который должен дать источник питания, \ $ \ text {I} \ $, равен \ $ \ text {I} _ {\ text {load}} \ $. Нам уже известно полное сопротивление цепи: \ $ \ text {R} = 250 + 2 \ cdot2.5 = 255 \ Омега \ $. Теперь мы можем рассчитать необходимое напряжение питания, используя \ $ \ text {V} = \ text {I} \ cdot {} \ text {R} \ $: \ $ \ text {V} = 0.02 \ cdot255 = 5.1 \ text { V} \ $


      Что, если мы хотим знать, сколько мощности теряется в проводах? Обычно мы используем \ $ \ text {P} = \ text {V} \ cdot {} \ text {I} \ $, где \ $ \ text {P} \ $ означает мощность, \ $ \ text {V} \ $ для напряжения и \ $ \ text {I} \ $ для тока.

      Итак, единственное, что нам нужно сделать, это ввести правильные значения в формулу.

      Пример

      Мы снова используем блок питания \ $ 5 \ text {V} \ $ с \ $ 250 \ Omega \ $ \ $ \ text {R} _ {\ text {load}} \ $ и двумя проводами \ $ 2.5 \ Omega \ $ за штуку. Падение напряжения на одном куске провода, как вычислено выше, составляет \ $ 0,049025 \ text {V} \ $. Ток в цепи был \ $ 0.01961 \ text {A} \ $.

      Теперь мы можем рассчитать потери мощности в одном проводе: \ $ \ text {P} _ {\ text {wire}} = 0,049025 \ cdot0.01961 = 0,00096138 \ text {W} = 0,96138 \ text {mW} \ $


      Во многих случаях нам известна длина провода \ $ l \ $ и AWG (американский калибр проводов) провода, но не сопротивление. Однако рассчитать сопротивление несложно.

      В Википедии есть список доступных здесь спецификаций AWG, который включает сопротивление на метр в Ом на километр или в миллиОм на метр. У них также есть килофуты или футы.

      Мы можем вычислить сопротивление провода \ $ \ text {R} _ {\ text {wire}} \ $, умножив длину провода на сопротивление на метр.

    Добавить комментарий

    Ваш адрес email не будет опубликован. Обязательные поля помечены *